You are on page 1of 101

XI MODULE - I

MOLE CONCEPT &


REDOX REACTION
Modus Operandi for Practice
(Suggested Ideal Approach)

Step – I CatalyseR 's Practice Sheets & NCERT


{while Chapter is running in Class-Room}
(Mandatory)

Step – II
CatalyseR 's Module
(Mandatory)

Step – III Previous Years’ JEE Subjective & Objective Questions,


R C Mukherjee solved and unsolved questions
(Mandatory)

Step – IV After Completion of Step – III, if time permits, students can


solve questions from previous year’s INCHO, NCHO, KVPY
(Optional) papers, Physical chemistry books by Atkins, Bruce H. Mahan

This Study Package is Prepared by


d`fr
o f Ca talyseR
t Wing
Conten
ELECTROSTATICS 1

MOLE CONCEPT
INDEX

 CONCEPTS IN BRIEF (MOLE CONCEPT) 01 – 13


 SOLVED EXAMPLES 14 – 18
 EXERCISE # 01 OBJECTIVE EXERCISE 19 – 24
 EXERCISE # 02 ADVANCED EXERCISE 25 – 34
 EXERCISE # 03 SUBJECTIVE EXERCISE (LEVEL # 01) 35 – 37
 EXERCISE # 04 SUBJECTIVE EXERCISE (LEVEL # 02) 38 – 41
 EXERCISE # 05 KVPY, OLYMPIADS QUESTIONS 42 – 43
 EXERCISE # 06 JEE (MAIN) CORNER 44 – 45
 EXERCISE # 07 JEE (ADVANCED) CORNER 46 – 47
 ANSWER KEYS 48 – 50

WEIGHTAGE OF ‘MOLE CONCEPT’ IN JEE (MAIN & ADVANCED) in Last Three Years

JEE (MAIN) Formely known as AIEEE

MARK /
YEAR No. Of Qs.
CHEMISTRY TOTAL MARKS
2015 1 4/120
2016 1 4/120
2017 2 8/120

JEE (ADVANCED)

MARK /
YEAR No. Of Qs.
CHEMISTRY TOTAL MARKS
2015 0 0/168
2016 1 3/124
2017 0 0/122

Nothing is impossible, the word itself says ‘I’m possible’!


MOLE CONCEPT 1

MOLE CONCEPT
INTRODUCTION

Anything that occupies space and has mass is called matter.

Ancient Indian and Greek Philospher’s beleived that the wide variety of object around us are
made from combination of five basic elements : Earth, Fire, Water, Air and Sky.

Ancient Greek Philosphers also believed that all matter was composed of tiny building blocks
which were hard and indivisible.

All these people have their philosphical view about matter, they were never put to experimental
tests, nor ever explain any scientific truth. It was John Dalton who firstly developed a theory on
the structure of matter, later on which is known as Dalton’s atomic theory.

DALTON’S ATOMIC THEORY:

1. Matter is made up of very small indivisible particle called atoms.


2. All the atoms of a given element is identical in all respect i.e. mass, shape, size, etc.
3. Atoms cannot be created or destroyed by any chemical process.
4. Atoms of different elements are different in nature.
5. Atoms combine in a definite proportion to form molecule

RELATIVE ATOMIC MASS :

1
It is the ratio of the mass of 1 atom of a substance and of mass of 1 atom of C12 isotope.
12
Dalton used hydrogen as the standard (H = 1). Later on oxygen (O = 16) replaced hydrogen as
the reference.
Relative atomic mass (R.A.M)
mass of one atom of the element

1
 mass of one C12 atom
12
total number of nucleons  massof 1 nucleon

1
12  massof 1 nucleon
12
 Total Number of nucleons

Don’t ever get comfortable when you have the ability to achieve more.
2 MOLE CONCEPT

ATOMIC MASS UNIT (OR AMU)

 1
The atomic mass unit (amu) is equal to one twelfth   of the mass of one atom of carbon-12
 12 
isotope.
1
 1 amu   mass of one C-12 atom  mass of one neucleon in C-12 atom.
12
 1.66  10 24 gm or 1.66  10 27 kg

THE MOLE

The term ‘mole’ is a Latin word which means heap or pile. A mole of atom is a collection of atoms
whose total weight is equal to the gram atomic weight. As equal number of moles of different
elements contain equal number of atoms, it is very easy to express the amount of a substance in
terms of moles. Just as a dozen means twelve objects, in the similar fashion, a mole is a
collection of a ‘definite number’ of particles, viz., atoms, molecules, ions or electrons. This
‘definite number’ is called Avogadronumber which is equal to 6.023  1023.
Thus, a mole can be defined as, “the amount of a substance containing as many atoms,
molecules, ions, electrons or other elementary entities as there are carbon atoms in exactly 12g
of C-12”.
Now-a-days, gram-molecules and gram-atom are termed as a mole of molecules and a mole
of atoms respectively, e.g., 1 gram-molecule of chlorine and 1 gram atom of chlorine are
expressed as 1 mole of Cl2 and 1 mole of Cl respectively.

Wt. of substancein g
No. of moles of molecules 
Mol. wt. of substance

Wt. of element ing


No. of moles of atoms 
At. wt. of the element

GRAM ATOMIC MASS :

The atomic mass of an element expressed in gram is called gram atomic mass of the element.
For example for oxygen atom :
Atomic mass of ‘O’ atom = mass of one ‘O’ atom = 16 amu
gram atomic mass = mass of 6.02  10 23 ‘O’ atoms
 16 amu  6.02  10 23  16  1.66  10 24 g  6.02  10 23  16g

 1.66  1024  6.02  1023  1 

CatalyseR Eduventures (India) Pvt. Ltd.


MOLE CONCEPT 3

GRAM MOLECULAR MASS :

The molecular mass of a substance expressed in gram is called the gram-molecular mass of the
substance.
or
23
It is also defined as mass of 6.02  10 molecules
or
It is also defined as the mass of one mole molecules. (molar mass)
For example for ‘O2’ molecule :
Molecular mass of ‘O2’ molecule = mass of one ‘O2’ molecule
= 2 × mass of one ‘O’ atom
= 2 × 16 amu = 32 amu
gram molecular mass = mass of 6.02  10 23 ‘O2’ molecules = 32 amu  6.02  1023

 32  1.66  10 24 gm  6.02  10 23  32gm

AVERAGE/ MEAN ATOMIC MASS :

The weighted average of the isotopic masses of the element’s naturally occuring isotopes.
Mathematically, average atomic mass of
a1x1  a 2 x 2  .....  an x n
XAx  
100

MEAN MOLAR MASS OR MOLECULAR MASS:

The average molar mass of the different substance present in the container
n1M1  n2M2  ......nnMn

n1  n2  ....nn

Don’t ever get comfortable when you have the ability to achieve more.
4 MOLE CONCEPT

GAY-LUSSAC’S LAW OF COMBINING VOLUME :

Gases combine in a simple ratio of their volumes provided all measurements should be done at
the same temperature and pressure
H2  g  + Cl2  g   2HCl

1 vol 1 vol 2 vol

AVOGADRO’S HYPOTHESIS :

Equal volume of all gases have equal number of molecules (not atoms) at same
temperature and pressure condition.

S.T.P. (Standard Temperature and Pressure):


At S.T.P. / N.T.P. condition :
temperature  0C or 273 K
pressure = 1 atm = 760 mm of Hg
volume of one mole of an ideal gas = 22.4 litres (experimentally determined)

THE LAW OF CHEMICAL COMBINATION

1. The law of conservation of mass :


In a chemical change total mass remains conserved.

2. Law of constant or Definite proportion :


All chemical compounds are found to have constant composition irrespective of their method of
preparation or sources.
Example :
In water  H2O , Hydrogen and Oxygen combine in 2 : 1 molar ratio, this ratio remains constant
whether it is tap water, river water or sea water or produced by any chemical reaction.

3. The law of multiple proportion :


When one element combines with the other element to form two or more different compounds,
the mass of one elements, which combines with a constant mass of the other, bear a simple ratio
to one another.

PERCENTAGE COMPOSITION AND MOLECULAR FORMULA :

Mass of element in 1 molof substance


%  100
Mass of 1 molof substance

CatalyseR Eduventures (India) Pvt. Ltd.


MOLE CONCEPT 5

EMPIRICAL AND MOLECULAR FORMULA :

The empirical formula of a compound is a chemical formula showing the relative number of atoms
in the simplest ratio, the molecular formula gives the actual number of atoms of each element in a
molecule. The molecular formula is generally an integral multiple of the empirical formula.
i.e. molecular formula  empirical formula  n

DENSITY :

It is of two type.
1. Absolute density 2. Relative density

For liquid and solids

mass
Absolute density 
volume
density of the substance
Relative density or specific gravity 
density of water at 4C

Vapour density is defined as the density of the gas with respect to hydrogen gas at the same
temperature and pressure.
dgas PMgas / RT Mgas Mgas
Vapour density   V.D.   Mgas  2  V.D.
dH2 PMH2 / RT MH2 2

CHEMICAL EQUATION :

All chemical reaction are represented by chemical equations by using chemical formule of
reactants and products.
 KCl  O (unbalanced chemical equation )
KClO3  2

 2KCl  3O (balanced chemical equation)


2KClO3  2

Remember a balnced chemical equation is one which contains an equal number atoms of each
element on both sides of equation.

INTERPRETATION OF BALANCED CHEMICAL EQUATIONS :

Once we get a balanced chemical equation then we can interpret a chemical equation by
following ways:
1. Mass - mass analysis
2. Mass - volume analysis
3. Mole - mole analysis
4. Vol - Vol analysis (separately discussed as eudiometry or gas analysis)

Don’t ever get comfortable when you have the ability to achieve more.
6 MOLE CONCEPT

1. Mass-mass analysis :

Consider the reaction


2KClO3 
 2KCl  3O2

According to stoichiometry of the reaction


mass-mass ratio: 2  122.5 : 2  74.5 : 3  32

Mass of KClO3 2  122.5


or 
Mass of KCl 2  74.5

Mass of KClO3 2  122.5



Mass of O2 2  32

2. Mass - volume analysis :

Now again consider decomposition of KClO3

2KClO 3  2KCl  3O 2

mass volume ratio : 2  122.5 gm : 2  74.5 gm : 3  22.4lt. at STP


we can use two relation for volume of oxygen
Mass of KClO3 2  122.5
 ...(i)
volume of O 2 at STP 3  22.4 lt

Mass of KCl 2  74.5


and  ...(ii)
volume of O 2 at STP 3  22.4 lt

3. Mole-mole Analysis :

This analysis is very much important for quantitative analysis point of view. Students are advised
to clearly understand this analysis.
Now consider again the decomposition of KClO3 .

2KClO3 
 2KCl  3O2

In very first step of mole-mole analysis you should read the balanced chemical equation like
2 moles KClO3 on decomposition gives you 2 moles KCl and 3 moles O2 .and from the
stoichiometry of reaction we can write
Moles of KClO3 Moles of KCl Moles of O 2
 
2 2 3

Now for any general balance chemical equation like a A + b B  c C + d D


you can write.
Mole of A reacted moles of B reacted moles of C produced moles of D produced
  
a b c d

CatalyseR Eduventures (India) Pvt. Ltd.


MOLE CONCEPT 7

PRINCIPLE OF ATOM CONSERVATION :

This principle is fruitful for the students when they don’t get the idea of balanced chemical
equation in the problem. This principle can be under stand by the following example.
Consider the decomposition of
KClO3  s  KCl  s  O2  g (unbalanced chemical reaction)

Apply the principle of atom conservation (POAC) for K atoms.


Moles of K atoms in reactant = moles of K atoms in products or moles of K atoms
in KClO3  moles of K atoms in KCl.

Now, since 1 molecule of KClO3 contains 1 atom of K or 1 mole of KClO3 contains 1 mole of K,
similarly,1 mole of KCl contains 1 mole of K.
Thus, moles of K atoms in KClO3  1 moles of KClO3 and moles of
K atoms in KCl  1  moles of KCl .
wt. of KClO3 in g wt. of KCl in g
  moles of KClO3  moles of KCl or 
mol. wt. of KClO3 mol. wt. of KCl

The above equation gives the mass-mass relationship between KClO3 and KCl which is
important in stoichiometric calculations.
Again, applying the principle of atom conservation for O atoms,
moles of O in KClO3  3  moles of KClO3 moles of O in O2  2  moles of O2

 3 × moles of KClO3

wt. of KClO3 vol. of O2 at NTP


 2  moles of O2 or 3   2
mol. wt. of KClO3 standard molar vol. (22.4 lt.)

SOME GENERAL TERMS

Specific gravity : It is the ratio of the density of a substance to that of water. (Relative density)

Specific volume : At a specified temperature and pressure, the volume occupied by unit mass of
a substance. (Reciprocal of density)

Dulong-Petit's Law : Sp. heat in calories x atomic mass = 6.4.

Trouton's Rule : The ratio of the molar latent heat of vapourisation to the Boiling point in Kelvin
is constant for all substances (The rule is only approximate)

Polymorphism : The existence of a given solid substance in more than one crystalline form is
known as polymorphism.
e.g: Quartz, Tridymite, Crystobalite are the three crystalline forms of Silica
SiO2 .

Don’t ever get comfortable when you have the ability to achieve more.
8 MOLE CONCEPT

Isomorphism : Different chemical units, if they have same crystalline form, it is said to be
isomorphism. e.g : Alums, K 2SO 4 & K 2SeO4 .

Allotropy : Polymorphism if confined to elements it is said to be allotropy.


e.g.: Diamond and graphite
Sulphur Rhombic and sulphur monoclinic.

Enantiotropy : Out of the two allotropic forms, one is stable below a particular temperature
and the other above. The transition of one form into other is reversible. Such a
one is said to be enantiotropy.
0
95.6
e.g.: SR   SM

Monotropy : If the transition is not reversible at a particular temperature, it is said to be


monotropy. The conversion is to be realised under suitable experimental
conditions.
e.g.: oxygen and ozone, Red phosphorus and yellow phosphorus
Diamond and graphite.

METHODS OF EXPRESSING THE CONCENTRATION OF A SOLUTION

The concentration of a solution can be expressed in a number of ways. The important methods are:

(i) Strength of solution:


Amount of solute present in one litre solution
weight of solute w
S 
volume of solution in litre V in (l )
 Normality  Equivalent weight
 Molarity  Molecular weight
(ii) Mass percentage or per cent by mass :
Mass of solute
%(w/w) Mass percentage of solute   100
Mass of solution
Mass of solute  100 Mass of solute  100
 
Mass of solute  Mass of solvent Volume of solution  Density of solution

(iii) Percent mass by volume :


Mass of solute
%(w/v)   100
Volume of soltuion

CatalyseR Eduventures (India) Pvt. Ltd.


MOLE CONCEPT 9

(iv) Parts per million (ppm) :


Mass of solute
ppm   10 6
Mass of solution

(v) Mole fraction :


Let n moles of solute (A) and N moles of solvent (B) be present in a solution.
n N
Mole fraction of solute   X A , Mole fraction of solvent   XB
Nn N n
In binary solution, X A  XB  1
Mole fraction is independent of temperature of the solution.

(vi) Molality
No. of moles of solute
Molality (m) =
weight (in kg) of solvent
Let wA grams of the solute of molecular mass mA be present in wB grams of the solvent, then
wA
Molality (m)   1000
mA  wB

Relation between mole fraction and Molality :

X A  1000 w  1000
m A
XB  mB wB  mB
Note : Molality is the most convenient method to express the concentration because it involves
the mass of liquids rather than their volumes. It is also independent of the variation in
temperature.

(vii) Molarity (Molar concentration)


No. of moles of solute
Molarity (M) 
Volume (in litre) of solution
wA
Molarity of the solution   1000
mA  V

Relation between molarity and % solute by mass :

Let d = density of solution in g/mL and let it contains x% (w/w) solute by mass.
Number of moles of solute in 1 litre
mass of solute in gram x  d  10 x  d  10
  M
grams molecular mass of solute Msolute mA
Molarity of mixing :
M1V1  M2 V2  M3 V3  MR  V1  V2  V3  MR  resultant molarity

Don’t ever get comfortable when you have the ability to achieve more.
10 MOLE CONCEPT

Relationship between molality and molarity :


molarity
Molality (m) 
d  molarity  msolute

1000  M

1000  d  M  Msolute

(viii) Normality :
It is define as no. of equivalent of a solute present in one litre of solution.
Equivalent of solute
N 
Volume of solution in litre
Weight of solute

Equivalent weight of solute  V in litre

w w  1000
N 
E  V in (l ) E  V in mL
Also Equivalent = N × V in l  H2 SO4 ,
and Milli equivalent = N × V in ml
Wt. of solute
=  1000
Eq. wt. of solute
Note : A striking fact regarding equivalent and milli equivalent is equivalent and milli
equivalent of reactants reacts in equal number to give same number of equivalent or milli
equivalent of products separately.

(ix) Formality :
Since molecular weight of ionic solids is not determined accurately experimentally due to
their dissociation nature and therefore molecular weight of ionic solid is often referred as
formula weight and molarity as formality.
Formality  SO3  H2O  H2SO 4 i.e., molarity

(x) Specific gravity of solution


 H2 SO 4 i.e., weight of 1 mL solution.

VOLUME STRENGTH OF H2 SO 4 SOLUTION

The concentration of H2 O2 is usually represented in terms of volume. If a sample of H2 O2 is


labeled as ‘x volume’, it means that 1 volume of H2 O2 solution gives ‘x volumes’ of O2 gas at STP on
complete decomposition.
Volume strength of H2 O2  5.6  Normality

CatalyseR Eduventures (India) Pvt. Ltd.


MOLE CONCEPT 11

PERCENTAGE LABELING OF OLEUM

Oleum or fuming sulphuric acid contains SO3 gas dissolved in sulphuric acid. When water is
added to oleum, SO3 reacts with H2O to form H2SO4 , thus mass of the solution increases.

SO3  H2O  H2SO4

The total mass of H2 SO 4 obtained by diluting 100g of sample of oleum with desired amount of
water, is equal to the percentage labeling of oleum.
 % labeling of oleum  Total mass of H2 SO 4 present in oleum after dilution.
 mass of H2 SO 4 initially present + mass of H2 SO 4 produced on dilution.

HARDNESS OF WAT ER

Temporary hardness : It is due to the presence of bicarbonates of calcium and magnesium and
it can be removed by heating or even by adding calculated quantities of
lime. It doesn’t give scum with soap.
Boil
Ca HCO3 2   CaCO3  H2O  CO2 ;
Boil
Ca HCO3 2  Ca  OH  2   2CaCO 3  2H2 O.

Permanent hardness : It is due to the presence of soluble salts of calcium and magnesium other
than bicarbonates.

i) Treatment with permutite


(Sodium Alumino Orthosilicate, Na2 Al2Si2 O8 .xH2O 

CaCl2  Na 2 Al2 Si2O 8  Ca  Al2 Si2 O8   2NaCl.

MgSO4  Na 2 Al2 Si2O 8  Mg  Al2Si2 O 8 2  Na 2 SO 4

ii) Treatment with Calgon (Sodium hexametaphosphate)

2Ca2  Na2 Na4  PO3 6   4Na  Na2 Ca2  PO3 6 

Degree of hardness : Hardness of water is calculated always in terms of calcium carbonate


although it is never responsible for causing hardness since it is an
insoluble one.
The hardness is expressed in terms of parts per million (ppm) and it is
the number of parts by weight of CaCO3 [Equivalent to calcium and

magnesium salts] present in a million 10


6
  parts by weight of water.

i.e. : 1ppm means 1 part of CaCO3 in 106 parts of water.

Don’t ever get comfortable when you have the ability to achieve more.
12 MOLE CONCEPT

CALCULATION OF AVAILABLE CHLORINE FROM A SAMPLE OF BLEACHNG POWDER–

The weight % of available Cl2 from the given sample of bleaching powder on reaction with dil
acids or CO2 is called available chlorine.
CaOCl2  H2 SO4  CaSO4  H2 O  Cl2

METHOD OF DETERMINATION OF AVAILABLE CHLORINE :


 Ca  CH 3COO  2  H 2O  Cl2
CaOCl2  2CH 3COOH 
(Sample of bleaching powder)

Cl2  2 KI 
 2 KCl  I 2
Starch as indicator
I 2  2 Na2 S 2 O3   Na2 S 4O6  2 Nal
v.f. = 2 v.f. = 1

End point is indicated by disappearance of blue colour.


Let M = Molarity of hypo Na 2S 2 O3  solution

 millimoles of Cl2 produced  milli moles of I 2 used by hypo

M V
 where V  vol of hypo solution used in ml.
2
mass of Cl2 produced

M  V  103
  71  35.5  M  V  10 3
2
 % of available chlorine
35.5  M  V  103
  100
W
where W  amount of belaching powder taken in gm.
3.55  M  V
or % of available Cl2 
W

CatalyseR Eduventures (India) Pvt. Ltd.


MOLE CONCEPT 13

LIMITING REAGENT :

The reactant which consumed first into the reaction


When you are dealing with balance chemical equation then if number of moles of reactants are
not in the ratio of stoichiometric coefficient of balalnced chemical equation, then there should be
one reactant which should be limiting reactant.

Ex. Three mole of Na2 CO3 is reacted with 6 moles of HCl solution. Find the volume of CO2 gas
produced at STP. The reaction is -
Na2 CO3  2HCl  2NaCl  CO2  H2O

Sol. From the reaction : Na2 CO3  2HCl  2NaCl  CO2  H2O

given moles 3 mol 6 mol


given mole ratio 1 : 2
Stoichiometric coefficient ratio 1 : 2
See here given moles of reactant are in stoichiometric coefficient ratio therefore none reactant left
over. Now use Mole-mole analysis to calculate volume of CO2 prdouced at STP

Moles of Na2 CO3 Mole of CO 2 Pr oduced


=
1 1
Moles of CO2 produced = 3 volume of CO2 produced at STP  3  22.4L  67.2L



Don’t ever get comfortable when you have the ability to achieve more.
14 MOLE CONCEPT

SOLVED EXAMPLES

Ex. 1 1.80 g of a certain metal burnt in oxygen gave 3.0 g of its oxide. 1.50 g of the same metal heated
in steam gave 2.50 g of its oxide. Show that these results illustrate the law of constant proportion.
Sol. In the first sample of the oxide, Wt. of metal = 1.80 g,
wt.of metal 1.80g
Wt. of oxygen = (3.0 – 1.80) g = 1.2 g    1.5
wt.of oxygen 1.2g
In the second sample of the oxide, Wt. of metal = 1.50 g, Wt. of oxygen = (2.50 – 1.50) g = 1 g.
wt.of metal 1.50g
   1.5
wt.of oxygen 1g
Thus, in both samples of the oxide the proportions of the weights of the metal and oxygen a fixed.
Hence, the results follow the law of constant proportion.

Ex. 2 An organic substance containing carbon, hydrogen and oxygen gave the following percentage
composition.
C = 40.687%; H = 5.085% and O = 54.228%
The molecular weight of the compound is 118. Calculate the molecular formula of the compound.
Sol.
Step-1 To calculate the empirical formula of the compound.

Element Symbol Percentage At. mass Relative no. Simplest Simplest whole
Percentage
of element of element of atoms = atomic ratio no. atomic ratio
At. mass

Carbon C 40.687 12 40.687 3.390


= 3.390 =1 2
12 3.389
5.085 5.085
Hydrogen H 5.085 1 = 5.085 =1.5 3
1 3.389
54.228 3.389
16 = 3.389 =1 2
Oxygen O 54.228 16 3.389

  Empirical Formula is C 2H3 O2

Step-2 To calculate the empirical formula mass.


The empirical formula of the compound is C 2H3 O2 .

  Empirical formula mass   2  12    3  1   2  16   59.

Molecular mass 118


Step-3 To calculate the value of ‘n’ n  2
Empirical formula mass 59

Step-4 To calculate the molecular formula of the salt.


Molecular formula  n  (Empirical formula)  2  C2H3 O2  C 4H6 O4

Thus the molecular formula is C4H6 O4 .

CatalyseR Eduventures (India) Pvt. Ltd.


MOLE CONCEPT 15

Ex. 3 367.5 gram KClO3 M  125.5  when heated. How many gram KCl and oxygen is produced.

Sol. Balance chemical equation for heating of KClO3 is

2KClO3 
 2KCl  3O2

mass-mass ratio : 2 × 122.5 gm : 2 × 74.5 gm : 3 × 32 gm


mass of KClO3 2  122.5

mass of KCl 2  74.5

367.5 122.5

W 74.5
W = 3 × 74.5 = 223.5 gm
Mass of KClO3 2  122.5
=
Mass of O 2 3  32

367.5 2  122.5
=
W 3  32
W = 144 gm

Ex. 4 The density of a 3 M sodium thiosulphate solution Na2S2O3  is 1.25 g/mL. Calculate

(i) the percentage by mass of sodium thiosulphate


(ii) the mole fraction of sodium thiosulphate
(iii) molalities of Na and S 2 O32  ions.

x  d  10 x  1.25  10
Sol. (i) M  3  x = 37.92
mA 158

474
(ii) No. of moles of Na 2S 2 O 3  3
158
Mass of water = (1250 – 474) = 776 g,
776
No.of moles of water   43.1
18
3 3
Mole fraction of Na 2 S 2O 3    0.065
43.1  3 46.1

(iii) No. of moles of Na ions  2  No. of moles of Na2 S2 O3  2  3  6

No. of moles of Na ions 6


Molality of Na ion    1000  7.73m
Mass of water in kg 776

No. of moles of S 2 O32  ions  No. of moles of Na 2S 2 O3

3
Molality of S 2 O32  ions   1000  3.86 m
776

Don’t ever get comfortable when you have the ability to achieve more.
16 MOLE CONCEPT

Ex. 5 A sample of H2 O2 is x% by mass. x ml of KMnO4 are required to oxidize one gram of this H2 O2
sample. Calculate the normality of KMnO4 solution.

Sol. Suppose, Mass of H2 O2 solution  100g


Mass of H2 O2 present  x gram
Mass of H2 O2 solution taken  1 gram

Mass of H2 O2 present in 1 gram solution  x


100
Equivalents of H2 O2  w
E
x
 …(A)
100  17

(E for H2 O2 )  17
x
Equivalents of KMnO 4  N  V  litre   N  x  10 3
100  17
Putting equivalents of H2 O2 and KMnO4 equal,

 N  x  103
N = 0.59 (Normality of KMnO4 )

Ex. 6 Calculate the composition of 109% oleum.


Sol. Let the mass of SO3 in the sample be ‘w’ g, then the mass of H2 SO 4 would be (100 – w)g.
On dilution,
SO3  H2O 
 H2 SO4

80g 18g
w
Moles of SO3 in oleum   Moles of H2 SO 4 formed after dilution.
80
98w
 Mass of H2 SO 4 formed on dilution 
80

Total mass of H2 SO 4 present in oleum after dilution

98w
  100  w   109
80

 w  40

Thus oleum sample contains 40% SO3 and 60% H2 SO 4 .

CatalyseR Eduventures (India) Pvt. Ltd.


MOLE CONCEPT 17

Ex. 7 One liter of sample of hard water contains 1mg of CaCl2 and 1 mg of MgCl2 . Find the total
hardness of water in terms of CaCO3 per 106 parts of water by weight

1 10 3  103 1  wt. 
Sol. m M of MgCl2    milli mole  1000 
95 95  M.wt 
1 10 3  10 3 1
m M of CaCl2  
111 111

 m M of CaCO3 if MgCl2 and CaCl2 are taken form of CaCO3

( Ca, Mg are both bivalent  mole ratio is 1 : 1)

1 1 206
  
95 111 111 95

Ex. 8 CS 2 and Cl2 in the weight ratio 1 : 2 are allowed to react according to equation find the fraction
of excess reagent left behind.
2CS 2  3Cl2 
 2CCl4  S 2Cl2

w 2w
Sol. mole
76 71
limiting reagent  CS 2

2 moles of CS 2 react with 3 mole of Cl2

1 mole of CS 2 react with 1.5 mole of Cl2

w w
mole of CS 2 react with 1.5  mole of Cl2
76 76
2w 3w
Mole of Cl2 left  
71 2 x 76

2w 3w

71 2 x 76
fraction of CS 2 left  x 100  0.299
2w
71

Don’t ever get comfortable when you have the ability to achieve more.
18 MOLE CONCEPT

Ex. 9 50.0 kg of N2  g and 10.0 kg of H2  g are mixed to produce NH3  g Calculate the NH3  g
formed. Identify the limiting reagent in the production of NH3 in this situation.
Sol. A balanced equation for the above reaction is written as follows :
Calculation of moles :
N2  g  3H2  g 2NH3  g

1000gN2 1molH2
moles of N2  50.0kg N2    17.86  102 mol
1kgN2 28.0gN2

1000gH2 1molH2
moles of H2  10.0kg H2    4.96  103 mol
1kgH2 2.016gH2

According to the above equation, 1 mol N2  g requires 3 mol H2  g , for the reaction, Hence, for

17.86  10 2 mol of N2 , the moles of H2  g required would be

3molH2 (g)
17.86  10 2 mol of N2   5.36  103 mol H2
1molN2 (g)

But we have only 4.96  103 mol H2 . Hence, dihydrogen is the limiting reagent in this case. So

NH3  g would be formed only from that amount of available digydrogen i.e., 4.96  103 mol

Since 3 mol H2  g gives 2 mol NH3  g

3molNH3 (g)
  4.96  103 mol H2  g   3.30  103 molNH3  g is obtained.
3 molH2 (g)
If they are to be converted to grams, it is done as follows :
17.0gNH3 (g)
3.30  103 molNH3  g   3.30  103  17gNH3  g 
1molNH3 (g)

 56.1 10 3 g NH3  56.1kgNH3

Ex. 10 Calculate the percentage of available chlorine in a sample of 3.55 g of bleaching powder which
was dissolved in 100 mL of water, 25 mL of this solution, on treatment with KI and dilute acid,
required 20 mL of 0.125 N sodium thiosulphate soqlution.
3.55  20  0.125
Sol. % of available Cl2   10%
3.55
 25
100



CatalyseR Eduventures (India) Pvt. Ltd.


MOLE CONCEPT 19

EXERCISE # 01 OBJECTIVE EXERCISE

1. In compound A, 1.00 g nitrogen unites with 0.57 g oxygen. In compound B, 2.00 g nitrogen
combines with 2.24 g oxygen. In compound C, 3.00 g nitrogen combines with 5.11 g oxygen.
These results obey the following law
(A) Law of constant proportion
(B) Law of multiple proportion
(C) Law of reciprocal proportion
(D) Dalton's law of partial pressure

2. 4.0 g of caustic soda (mol mass 40) contains same number of sodium ions as are present in-
(A) 10.6 g of Na 2 CO3 (mol. mass 106)

(B) 58.5 g of NaCl (Formula mass 58.5)


(C) 100 ml of 0.5 M Na 2 SO 4 (Formula mass 142)

(D) 1mol of NaNO3 (mol. mass 85)

3. No. of oxalic acid molecules in 100 ml of 0.02 N oxalic acid is -


(A) 6.023  10 20 (B) 6.023  10 21 (C) 6.023  10 22 (D) 6.023  10 23

4. One atom of an element ‘X’ weighs 6.664  10  23 gm. The number of gram atoms in 40 kg of it is-
(A) 10 (B) 100 (C) 10000 (D) 1000

5. The sulphate of a metal M contains 9.87% of M. This sulphate is isomorphous with ZnSO 4 .7H2 O .
The atomic weight of M is
(A) 40.3 (B) 36.3 (C) 24.3 (D) 11.3

6. Caffeine has a molecular weight of 194. If it contains 28.9% by mass of nitrogen, number of
atoms of nitrogen in one molecule of caffeine is
(A) 4 (B) 6 (C) 2 (D) 3

7. 1.520 g of the hydroxide of a metal on ignition gave 0.995 gm of oxide. The equivalent weight of
metal is
(A) 1.520 (B) 0.995 (C) 19.00 (D) 9.00

8. 5.6 litre of a gas at N.T.P. weighs equal to 8 gm the vapour density of gas is -
(A) 32 (B) 16 (C) 8 (D) 40

9. Number of gm of oxygen in 32.2 g Na2 SO 4 .10H2 O is:


(A) 20.8 (B) 22.4 (C) 2.24 (D) 2.08

Don’t ever get comfortable when you have the ability to achieve more.
20 MOLE CONCEPT

10. A sample of phosphorus trichloride (PCl3 ) contains 1.4 moles of the substance. How many atoms
are there in the sample
(A) 4 (B) 5.6
(C) 8.431  10 23 (D) 3.372  10 24

11. The percent of N in 66% pure (NH4)2 SO4 sample is -


(A) 32 (B) 28 (C) 14 (D) None of these

12. Equal weight of 'X' (At. wt. = 36) and 'Y' (At. wt.= 24) are reacted to form the compound X 2 Y3 .
Then
(A) X is the limiting reagent (B) Y is the limiting reagent
(C) No reactant is left over and mass of X2 Y3 formed is double the mass of ‘X’ taken
(D) None of these

13. How much of NaOH is required to neutralise 1500 cm 3 of 0.1 NHCl (Na = 23)
(A) 40 g (B) 4g (C) 6g (D) 60 g

14. What will be the volume of CO2 at NTP obtained on heating 10 grams of (90% pure) limestone
(A) 22.4 litre (B) 2.016 litre (C) 2.24 litre (D) 20.16 litre

15. The volume of water that must be added to a mixture of 250 ml of 0.6 M HCl and 750 ml of 0.2 M
HCl to obtain 0.25 M solution of HCl is :
(A) 0.75 (B) 100 ml (C) 200 m (D) 300 m

16. What volume of 0.10 M H2SO4 must be added to 50 mL of a 0.10 M NaOH solution to make a
solution in which the molarity of the H2SO4 is 0.050 M?
(A) 400 mL (B) 50 mL (C) 100 mL (D) 150 mL

17. 500 mL of a glucose solution contains 6.02 × 1022 molecules. The concentration of the solution is
(A) 0.1 M (B) 1.0 M (C) 0.2 M (D) 2.0 M

18. Mole fraction of A in H2O is 0.2. The molality of A in H2O is :


(A) 13.8 (B) 15.5 (C) 14.5 (D) 16.8

19. What is the molarity of H2SO4 solution that has a density of 1.84 g/cc and contains 98% by mass
of H2SO4. (Given atomic mass of S = 32)
(A) 4.18 M (B) 8.14 M (C) 18.4 M (D) 18 M

CatalyseR Eduventures (India) Pvt. Ltd.


MOLE CONCEPT 21

20. A hydrocarbon contains 86% carbon, 488ml of the hydrocarbon weight 1.68 g at STP. Then the
hydrocarbon is an
(A) Alkane (B) Alkene (C) Alkyne (D) Arene

21. 16.8 litre gas containing H2 and O2 is formed at NTP on electrolysis of water. What should be the
weight of electrolysed water
(A) 5g (B) 9g (C) 10 g (D) 12 g

22. 100 g CaCO3 reacts with 1litre 1 N HCl. On completion of reaction how much weight of CO2 will
be obtain
(A) 5.5 g (B) 11 g (C) 22 g (D) 33 g

23. 4.4 g of an unknown gas occupies 2.24L of volume at standard temperature and pressure. The
gas may be
(A) Carbon dioxide (B) Carbon monoxide
(C) Oxygen (D) Sulphur dioxide

24. The equivalent weight of a metal is 9 and vapour density of its chloride is 59.25. The atomic
weight of metal is
(A) 23.9 (B) 27.3 (C) 36.3 (D) 48.3

25. Under similar conditions of pressure and temperature, 40 ml of slightly moist hydrogen chloride
gas is mixed with 20 ml of ammonia gas, the final volume of gas at the same temperature and
pressure will be
(A) 100 ml (B) 20 ml (C) 40 ml (D) 60 ml

26. 1 gram molecule of chlorine combines with a certain weight of a metal giving 111 g of its chloride.
The same weight of the metal displaced 2 g of hydrogen from an acid. The atomic weight of the
metal (assuming its valency to be 2) is :
(A) 40 (B) 20 (C) 80 (D) none of these

27. A certain compound has the molecular formula X4O6 . If 10 gm of compound contains 5.62 g of X,
the atomic mass of X :
(A) 32 amu (B) 37 amu
(C) 42 amu (D) 48 amu

28. What weight of CuSO 4 .5H2O must be taken to make 0.5 litre of 0.01 0.01M Cu2  ions solution?
(A) 12.45 gm (B) 1.245 gm
(C) 6.24 gm (D) 100 gm

Don’t ever get comfortable when you have the ability to achieve more.
22 MOLE CONCEPT

29. A partially dried clay mineral contains 8% water. The original sample contained 12 % water and
45 % silica. The % of silica in the partially dried sample is nearly :
(A) 50 % (B) 49 % (C) 55 % (D) 47 %

30. 8 g of sulphur is burnt to form SO2 which is oxidised by Cl2 water. The solution is treated with
BaCl2 solution. The amount of BaSO4 precipitated is :

(A) 1 mole (B) 0.5 mole (C) 0.24 mole (D) 0.25 mole

31. 1 gram of a carbonate of a metal was dissolved in 25 ml of 1N HCl. The resulting liquid required 5
ml of N NaOH for neutralization. The eq. wt. of metal carbonate is :
(A) 100 (B) 30 (C) 40 (D) 50

32. Sulphuryl chloride SO2Cl2 reacts with water to give a mixture of H2 SO 4 and HCl. How many
moles of NaOH would be needed to neutralize the solution formed by adding 1 mole of SO2Cl2 to
excess of water.
(A) 1 (B) 3 (C) 2 (D) 4

33. One mole of a mixture of CO and CO2 requires exactly 20 gram of NaOH in solution for complete
conversion of all the CO2 into Na 2CO3 . How many grams more of NaOH would it require for
conversion into Na 2 CO3 if the mixture (one mole) is completely oxidized to CO2 .
(A) 60 grams (B) 80 grams
(C) 40 grams (D) 20 grams

34. One gram of a variable alloy of lead and tin yielded on treatment with conc. HNO3, filtration and
heating 0.42 grams of SnO2. If all the lead in one gram of alloy is converted into PbSO4, what will
be the weight of PbSO4 obtained?

(A) 0.98 (B) 1g


(C) 1.07 g (D) 0.95 g

35. 34 g of hydrogen peroxide is present in 1120 ml of solution. This solution is called :


(A) 10 vol solution (B) 20 vol solution
(C) 30 vol solution (D) 32 vol solution

36. Exactly 4.00 gm of a solution of H2SO4 was diluted with water and excess BaCl2 was added.
The washed and dried precipitate of BaSO4 weighed 4.08 gm. Find the percent H2SO4 in the
original acid solution.
(A) 43.0 % (B) 4.3 % (C) 47 % (D) none

CatalyseR Eduventures (India) Pvt. Ltd.


MOLE CONCEPT 23

37. One gram of the silver salt of an organic dibasic acid yields, on strong heating, 0.5934 g of silver.
If the weight percentage of carbon in it 8 times the weight percentage of hydrogen and one-half
the weight percentage of oxygen, determine the molecular formula of the acid. [Atomic weight of
Ag = 108]
(A) C4H6O4 (B) C4H6O6 (C) C2H6O2 (D) C5H10O5

38. Mass of sucrose C12H22O11 produced by mixing 84 gm of carbon, 12 gm of hydrogen and


56 lit. O2 at 1 atm & 273 K according to given reaction, is

C(s) + H2(g) + O2 (g)  C12H22O11(s)

(A) 138.5 (B) 155.5 (C) 172.5 (D) 199.5

39. 40 gm of a carbonate of an alkali metal or alkaline earth metal containing some inert impurities
was made to react with excess HCl solution. The liberated CO2 occupied 12.315 lit. at 1 atm &
300 K. The correct option is
(A) Mass of impurity is 1 gm and metal is Be (B) Mass of impurity is 3 gm and metal is Li
(C) Mass of impurity is 5 gm and metal is Be (D) Mass of impurity is 2 gm and metal is Mg

40. The percentage by mole of NO2 in a mixture of NO2(g) and NO(g) having average molecular
mass 34 is :
(A) 25% (B) 20% (C) 40% (D) 75%

41. The minimum mass of mixture of A2 and B4 required to produce at least 1 kg of each product is :

(Given At. mass of 'A' = 10 ; At. mass of 'B' = 120)


5A2 + 2B4  2AB2 + 4A2B

(A) 2120 gm (B) 1060 gm (C) 560 gm (D) 1660 gm

42. 74 gm of a sample on complete combustion gives 132 gm CO2 and 54 gm of H2O. The
molecular formula of the compound may be
(A) C5H12 (B) C4H10O (C) C3H6O2 (D) C3H7O2

43. An iodized salt contains 0.5 % of NaI. A person consumes 3 gm of salt everyday. The number of
iodide ions going into his body everyday is

(A) 10–4 (B) 6.02 ×10–4 (C) 6.02 × 1019 (D) 6.02 × 1023

44. The mass of CO2 produced from 620 gm mixture of C2H4O2 & O2, prepared to produce
maximum energy is (Combustion reaction is exothermic)
(A) 413.33 gm (B) 593.04 gm (C) 440 gm (D) 320 gm

Don’t ever get comfortable when you have the ability to achieve more.
24 MOLE CONCEPT

45. In the quantitative determination of nitrogen, N2 gas liberated from 0.42 gm of a sample of
100
organic compound was collected over water. If the volume of N2 gas collected was ml at
11
total pressure 860 mm Hg at 250 K, % by mass of nitrogen in the organic compound is

[Aq. tension at 250 K is 24 mm Hg and R = 0.08 L atm mol–1 K–1]


10 5 20 100
(A) % (B) % (C) % (D) %
3 3 3 3

46. The mass of P4 O10 produced if 440 gm of P4 S3 is mixed with 384 gm of O2 is :

P4 S 3  O 2  P4 O10  SO2


(A) 568 gm (B) 426 gm (C) 284 gm (D) 396 gm

47. Calculate percentage change in Mavg of the mixture, if PCl5 undergo 50% decomposition.

PCl5  PCl3  Cl2


(A) 50% (B) 66.66 % (C) 33.33 % (D) Zero

48. The mass of Mg3N2 produced if 48 gm of Mg metal is reacted with 34 gm NH3 gas is

Mg + NH3  Mg3N2 + H2

200 100 400 150


(A) (B) (C) (D)
3 3 3 3

49. The number of carbon atoms present in a signature, if a signature written by carbon pencil
weights 1.2  103 g is:

(A) 12.04  10 20 (B) 6.02  1019 (C) 3.01  1019 (D) 6.02  10 20

24
50. The average atomic mass of a mixture containing 79 mole % of Mg and remaining 21 mole %
25 26 26
of Mg and Mg, is 24.31. % mole of Mg is:

(A) 5 (B) 20 (C) 10 (D) 15



CatalyseR Eduventures (India) Pvt. Ltd.


MOLE CONCEPT 25

EXERCISE # 02 ADVANCED LEVEL EXERCISE


(COMPREHENSION TYPE, MULTIPLE TYPE, MATRIX MATCH TYPE, INTEGER TYPE)

MULTIPLE ANSWERS CORRECT TYPE

1. Which of the statements are true?


(A) The equivalent weight of Ca3 PO 4 2 is Mw/6.

(B) The equivalent weight of Na3PO4 .12H2O is Mw/3.

(C) The equivalent weight of K 2 SO4 is Mw/2.

(D) The equivalent weight of potash alum K 2SO4 Al2  SO 4 3 .24H2 O is Mw/8.

Where Mw is the molecular weight of the respective compounds.

2. Which of the following statements is/are correct?


(A) CaC 2 is made in an electric furnace by the reaction: CaO  C 
 CaC2  CO . 16.0 g of
CaC 2 is obtained from 9.0 g of C.

(B) Polyethene can be produced form CaC2 as follows:

CaC 2  H2 O 
 CaO  HC  CH

HC  CH  H2  H2 C  CH2

n  CH2  CH2  
  CH2  CH2  n (Polyethene)

32.0 kg of CaC2 produces 14.0 kg of polyethene.

(C) 1.435 g of AgCl is obtained from 17.75 g of  Ag  NH3 2  Cl by the following reaction:

 Ag NH3 2  Cl  2HNO 3  AgCl  2NH 4 NO 3

(D) Commercial sodium ‘hydrosulfite’ is 50% pure Na2 S 2 O 4 . It is prepared as follows:

i. Zn  2SO2 
 ZnS2 O4

ii. ZnS 2 O4  Na2 CO3  ZnCO3  Na 2 S 2O 4

174.0 metric ton of commercial product Na2S2O4  can be made from 65.4 metric ton of Zn, with
a sufficient supply of other reactants.

3. Which of the following statements is/are correct?


1.0 g mixture of CaCO3  s  and glass beads liberate 0.22 g of CO2 upon treatment with excess
of HCl. Glass does not react with HCl.
CaCO3  2HCl  CO 2  H2O  CaCl2

[Mw CaCO 3  100, Mw of CO 2  44 , [Atomic weight of Ca = 40]

Don’t ever get comfortable when you have the ability to achieve more.
26 MOLE CONCEPT

(A) The weight of CaCO3 in the original mixture is 0.5 g.


(B) The weight of calcium in the original mixture is 0.2 g.
(C) The weight percent of calcium in the original mixture is 40% Ca.
(D) The weight percent of Ca in the original mixture is 20% Ca.

4. Which of the following statements is/are correct?


Commercial HCl is prepared by heating NaCl with H2 SO 4 :

2NaCl  H2SO 4  Na2 SO4  2HCl

(A) 196.0 g of pure H2 SO 4 is required for the production of 245.0 g of conc HCl containing 40%
HCl by weight.
(B) 245.0 g of 80% H2 SO 4 by weight is required for the production of 365.0 g of conc. HCl
containing 40% HCl by weight.
(C) 2 mol of pure H2 SO 4 is required for the production of 365 g of 40% HCl.

(D) 2.5 mol of 80% H2 SO 4 is required for the production of 365.0 g of 40% HCl.

5. Which of the following statements is/are wrong? The following reactions occur:
i. P4  5O2 
 P4 O10

ii. P4  3O 2  P4 O 6

1.24 g of P4 reacts with 8.0 g of O2

(A) P4 is the limiting quantity(B) O2 is the limiting quantity

(C) Mass of P4 O10 obtained is 2.2 g (D) Mass of P4 O6 obtained is 2.84 g

6. Which of the following statements is/are correct?


i. 21.0 g of lithium reacts with 32.0 g of O2 .

4Li  O2  2Li2 O

ii. 3.9 g of K reacts with 4.26 g of Cl2

2K  Cl2 
 2KCl

[Atomic weight of Li = 7 and K = 39. Mwof Li2 O  30 and KCl = 74.5 g mol1 )

(A) In reaction (i), O2 is in excess. (B) 45.0 g of Li2O is formed is reaction (i)

(C) In reaction (ii), Cl2 is in excess (D) 7.45 g of KCl is formed is reaction (ii)

7. Which of the following have equal mass of Cl ions in 1.0 L of each of the following solutions?
(A) 5% NaCl (density = 1.07 g mL1 ) (B) 5% KCl (d = 1.06 g mL1 )
(C) 58.5 g NaCl (D) 55.5 g BaCl2

CatalyseR Eduventures (India) Pvt. Ltd.


MOLE CONCEPT 27

8. Which of the following statements is/are correct?


20.0 mL of 6.0 M HCl is mixed with 50.0 mL of 2.0 M Ba  OH 2 , and 30mL of water is added.

(A) The concentration of OH remaining in solution is 0.8 M.
(B) The concentration of Cl remaining in solution is 1.2 M.
(C) The concentration of Ba 2  remaining in solution is 1.0 M.

(D) 80 m moles of OH is in excess.

9. An excellent solution for cleaning grease stains from cloth or leather consists of the following
components: CCl4 (80% by volume), ligroin (16%), and amyl alcohol (4%). How many mL of each
should to taken to make up 80 mL of solution?
(A) 64 mL CCl4
(B) 12.8 mL ligroin
(C) 32 mL of amyl alcohol
(D) 3.2 mL of amyl alcohol

10. If 100 mL of 1 M H2 SO 4 solution is mixed with 100 mL of 98% (W/W) of H2 SO 4 solution


 d  0.1 g mL  , then
1

(A) Concentration of solution becomes half.


(B) Volume of solution becomes 200 mL.
(C) Mass of H2 SO 4 in the solution is 98 g.

(D) Mass of H2 SO 4 in the solution is 19.6 g.

11. When 100 mL of 0.1 M KNO3 and 400 mL of 0.2 M HCl and 500 mL of 0.3 M H2 SO 4 are mixed,
then in the resulting solution
(A) The molarity of K   0.01 M
(B) The molarity of SO 24   0.15 M

(C) The molarity of H  0.38 M


(D) The molarity of NO3  0.08 and Cl  0.01 M

12. 1 mol BaF2  2molH2S 4  resulting mixture will be neutralized by :


(A) 1 mol of KOH
(B) 2 mol of Ca  OH 2

(C) 4 mol KOH


(D) 2 mol of KOH

Don’t ever get comfortable when you have the ability to achieve more.
28 MOLE CONCEPT

13. Given following series of reactions:


(I) NH3  O 2  NO  H2O

(II) NO  O2  NO2

(III) NO2  H2O  HNO3  HNO2

(IV) HNO2 
 HNO 2  NO  H2O

Select the correct option(s):


(A) Moles of HNO3 obtained is half of moles of Ammonia used if HNO2 is not used to produce
HNO3 by reation (IV)

100
(B) % more HNO3 will be produced if HNO2 is used to produce HNO3 by reaction (IV)
6
than if HNO2 is not used to produce HNO3 by reaction (IV)

1 th
(C) If HNO2 is used to produce HNO3 then of total HNO3 is produced by reaction (IV)
4
(D) Moles of NO produced in reaction (IV) is 50% of moles of total HNO3 produced.

14. 2 mol of CO2 is required to prepare


(A) 336 of NaHCO3 (B) 168 g of NaHCO3

(C) 463 g of Ca HCO3 2 (D) 162 g of Ca HCO3 2

15. 50 milliliters of CO is mixed with 20 mL of oxygen and sparked. After the reaction, the mixture is
treated with an aqueous KOH solution. Choose the correct option.
(A) The volume of the CO that reacts = 40 mL
(B) The volume of the CO2 formed = 40 mL.
(C) The volume of the CO that remains after treatment with KOH = 10 mL
(D) The volume of the CO that remains after treatment with KOH = 20 mL

16. ‘20 volumes’ of H2 O2 is equal to :


(A) 20% H2 O2 by mass (B) 6% H2 O2 by mass
(C) 1.764 N (D) 3.528 N

17. 10.78 g of H3PO4 in 550 ml solution is 0.40 N. Thus this acid :


(A) has been neutralised to HPO24 (B) has been neutralized to PO 24 

(C) has been reduced to HPO23  (D) has been neutralised to H2PO4

CatalyseR Eduventures (India) Pvt. Ltd.


MOLE CONCEPT 29

COMPREHENSION TYPE
PARAGRAPH - I

Cisplatin is used as an anticancer agent for the treatment of solid tumors, and its prepared as
follows:
K 2 PtCl4   2NH3  Pt NH3  2 Cl2   2KCl

Potassium tetra Cisplatin


Chloro platinate (II)
Given 83.0 g of K 2 PtCl4  is reacted with 83.0 g of NH3 .

[Atomic weights: K = 39, Pt = 415, Cl = 35.5, N = 14]

18. Which reactant is the limiting reagent and which is in excess?


Limiting Excess
(A) K 2 PtCl4  NH3

(B) NH3 K 2 PtCl4 

(C) None None


(D) Both Both

19. The number of mol of K 2 PtCl4  and NH3 used, respectively, are
(A) 0.1, 0.2 (B) 0.2, 0.4 (C) 0.3, 0.6 (D) 0.03, 0.06

20. The number of mol of excess reactant is


(A) 4.68 (B) 4.78 (C) 4.58 (D) 4.48

PARAGRAPH – II

When phosphours P4  is heated in limited amount of O2 ,P4 O6 (tetraphosphorus hexaoxide) is obtained,
and in excess of O 2 ,P4 O10 (tetraphosphours decaoxide) is obtained.

i. P  3O2  P4 O6 ;
ii. P4  5O2  P4 O10

21. What mass of P4 O6 will be produced by the combustion of 2.0 g of P4 with 2.0 g of O2 .
(A) 0.0145 mol (B) 0.016 mol (C) 0.029 (D) 0.0048

22. What mass of P4 O10 will be produced by the combustion of 2.0 g of P4 with 2.0 g of O2 .
(A) 1.04 g (B) 0.52 g (C) 2.04 g (D) 3.04 g

23. How many moles of O2 left unreacted initially in reaction (i)?


(A) 0.0145 mol (B) 0.072 mol (C) 0.029 mol (D) 0.0048 mol

Don’t ever get comfortable when you have the ability to achieve more.
30 MOLE CONCEPT

PARAGRAPH – III

A mixture of a mol of C3H8 and b mol of C2H4 was kept is a container of V L exerts a pressure of 4.93 atm
at temperature T. Mixture was burnt in presence of O2 to convert C3H8 and C2H4 into CO2 in the
container at the same temperature. The pressure of gases after the reaction and attaining the thermal
equilibrium with atmosphere at temperature T was found to be 11.08 atm.

24. The mole fraction of C3H8 in the mixture is


(A) 0.25 (B) 0.75
(C) 0.45 (D) 0.55

25. The mole fraction of C2H4 in the mixture is


(A) 0.25 (B) 0.75
(C) 0.45 (D) 0.55

26. The moles of O2 needed for combustion at temperature T is equal to


(A) 14 a (B) 14 b
(C) 15 a (D) 12 b

PARAGRAPH – IV

The term volume strength is used to express strength of H2O2. The strength of ‘XV’ means 1V
H2O2 on decomposition gives X volume of O2 at STP. In other words, 1 L of ‘XV’ H2O2
Produces X L of O2 at STP. H2O2 (aq.) decompose to H2O(l) and O2(g).

27. The percentage strength (w/v) of "11.2 V" H2O2 will be -


(A) 1.7 (B) 3.4
(C) 34 (D) None of these

28. 5. 40 g of Ba(MnO4)2 (mol. mass = 375 g mol–1) sample containing some inert impurity in
acidic medium is completely reacted with 125 mL of "33.6 V" of H2O2. The percentage
purity of the sample is -
(A) 28.12 (B) 70.31
(C) 85 (D) None of these

29. 6. 20 mL of H2O2 solution needs 80 mL of 0.05 M KMnO4 in acidic medium. The volume
strength of H2O2 is-
(A) 11.2 (B) 5.6
(C) 2.8 (D) 1.4

CatalyseR Eduventures (India) Pvt. Ltd.


MOLE CONCEPT 31

PARAGRAPH – V

Preparation of cobalt Metaborate involves the following steps of reactions:

(i) Ca2B6O11 + Na2CO3 (aq)  CaCO3 (insoluble) + Na2B4O7 + 2NaBO2

(ii) Na2B4O7  NaBO2 + B2O3

(iii) CoO + B2O3  Co(BO2)2.

30. Mass of Ca2B6O11 in kg required to produce 14.5 kg of Co(BO2)2 , assuming 100% yield of
each reaction is
(A) 32.2 (B) 40 (C) 28.2 (D) 30

200
31. If the yield of reaction (i), (ii) & (iii) is 60%, % & 32.2 % respectively, then mass of
3
Ca2B6O11 in kg required to produce 14.5 kg ofCo(BO2)2 is
(A) 250 (B) 200 (C) 190 (D) 150

PARAGRAPH – VI

A 4.925 g sample of a mixture of CuCl2 and CuBr2 was dissolved in water and mixed thoroughly
with a 5.74 g portion of AgCl. After the reaction the solid, a mixture of AgCl and AgBr, was
filtered, washed, and dried. Its mass was found to be 6.63 g.

32. % By mass of CuBr2 in original mixture is


(A) 2.24 (B) 74.5
(C) 45.3 (D) None

33. % By mass of Cu in original mixture is


(A) 38.68 (B) 19.05
(C) 3.86 (D) None

34. % by mole of AgBr in dried precipate is


(A) 25 (B) 50
(C) 75 (D) 60

35. No. of moles of Clr ion present in the solution after precipitation are
(A) 0.06 (B) 0.02
(C) 0.04 (D) None

Don’t ever get comfortable when you have the ability to achieve more.
32 MOLE CONCEPT

PARAGRAPH – VII

Water is added to 3.52 grams of UF6 . The products are 3.08 grams of a solid [containing only U,
O and F] and 0.8 gram of a gas only. The gas [containing fluorine and hydrogen only], contains
95 % by mass fluorine.
[Assume that the empirical formula is same as molecular formula.]

36. The empirical formula of the gas is


(A) HF2 (B) H2F (C) HF (D) HF3

37. The empirical formula of the solid product is


(A) UF2 O2 (B) UFO2 (C) UF2 O (D) UFO

38. The percentage of fluorine of the original compound which is converted into gaseous compound
is
(A) 66.66 % (B) 33.33 % (C) 50 % (D) 89.9 %

MATCH THE COLUMN TYPE


39. Match the reactions given in column I with molarities given in column II.
Column I Column II

a. What molar concentration of K 4 Fe  CN  6  should be used so that 40 p. 0.25 M

mL of a solution titrates 196.2 mg Zn (dissolved) by forming


K 2 Zn3 Fe  CN 6  (Atomic weight Zn  65.4 gmol1 )
2

b. 40.0 mL of Na 2 SO 4 solution is treated with an excess of BaCl2 . If the q. 0.05 M


mass of the precipitated BaSO4 is 2.33 g, the molarity of the Na 2 SO 4
solution is (Mw of BaSO4 = 233 g mol1 )

BaCl2  Na 2SO 4  BaSO 4  2NaCl

c. A sample of solution containing 116 mg of Th 4  requires 50.0 mL of r. 0.02 M


H2 C 2 O 4 to precipitate Th  C2 O4 2

Th4   2C 2O 42 
 Th  C2 O 4 2

What is the molarity of H2 C 2 O 4 (Atomic weight of Th = 232 g)

d. 10 mL of 0.5 M AgNO3 was required to react exactly with 100 mL s. 0.1 M


solution of NaCN.

Ag  2CN  Ag  CN  2

What is the molarity of NaCN

CatalyseR Eduventures (India) Pvt. Ltd.


MOLE CONCEPT 33

40. Match the reactions given in column I with neutralization reactions given in column II.

Column I Column II

a. 0.1 mol Na 2 CO3 p. 320 mL of 0.25 N KOH solution

0.2 mol NaHCO3

0.3molNaCl

b. 200 mL of 0.1 M HCl q. 400 mL of 0.5 M H2 SO 4


+100 of 0.1 M H2 SO 4

+200 mL of 0.1 M H2 C 2 O 4

c. 1g NaOH and 2.25 g of oxalic acid r. 125 mL of N/5 Mg  OH 2

d. 0.01 mol H3PO4 and 0.0025 mol of Ca  OH 2 s. 125 mL of N/5 H2 SO 4

41. The recommended daily dose is 17.6 milligrams of vitamin C (ascorbic acid) having formula.
C6H8O6. Match the following. Given : NA = 6 × 1023

Column I Column II
a. O-atoms present p. 10–4 mole
b. Moles of vitamin C in 1 gm of vitamin C q. 5.68 × 10–3

c. Moles of vitamin C in 1 gm should be consumed daily r. 3.6 × 1020

42. One type of artifical diamond (commonly called YAG for yttrium aluminium garnet) can be
represented by the formula Y3 Al5 O12 .  Y  89, Al  27 

Column I Column II
(Element) (Weight percentage)
(A) Y (P) 22.73%
(B) Al (Q) 32.32%
(C) O (R) 44.95%

Don’t ever get comfortable when you have the ability to achieve more.
34 MOLE CONCEPT

INTEGER TYPE

43. What is the volume of water in ml of 3.011  10 23 molecule of water? (d = 1g/ml)

44. A sample of chlorine has only two isotopes, Cl35 and Cl37 and its average atomic weight is 35.82.
the percentage abundance of isotope Cl37 in the given sample is.

45. Atomic weight of Al on a non-conventional scale is found to be 18 while on conventional scale, it


 1  th
is 27. On the new scale an amu is defined as   part by weight of C-12. Here X is:
x

46. If a protein has 0.07% Fe (M = 56) by weight as the only metal, its molar mass would be at least
M × 104 g. Here M is.

47. On heating 1.763 g of hydrated barium chloride BaCl2  H2 O to dryness, 1.505 g of anhydrous
salt remained. Find the value of x (Mol. wt. of BaCl2  208 ) -

48. Find the number of g-molecules of oxygen in 6.023  10 24 CO molecules -

49. If a mixture containing 3 moles of hydrogen and 1 mole of nitrogen is converted completely into
ammonia the ratio of initial and final volumes under the same temperature and pressure would be

50. The formula of a gas is [CO]x. If vapour density is 70, the value of x will be.



CatalyseR Eduventures (India) Pvt. Ltd.


MOLE CONCEPT 35

EXERCISE # 03 SUBJECTIVE EXERCISE (LEVEL # 01)

1. A plant virus is found to consist of uniform symmetrical particles of 150 Å in diameter and 5000 Å
long. The specific volume of the virus is 0.75 cm3/g. If the virus is considered to be a single
particle, find its molecular weight.

2. If all 1 billion 10 9  people in India were put to work counting the atoms in a mole of gold and if
each person could count one atom per second day and night for 365 days a year, how many
years would it take to finish the count?

3. Vitamin C, ascorbic acid, has the formula C 6H8 O 6 .


(a) The recommended daily dose of vitamin C is 60 milligrams. How many moles are you consuming
if you ingest 60 mg of the vitamin?
(b) A typical tablet contains 1.00 g of vitamin C. How many moles of vitamin C does this represent?
(c) When you consume 1.00 gram of vitamin C, how many oxygen atoms are you eating?

4. The action of bacteria on meat and fish produces a poisonous compound called cadaverine. As
its name and origin imply, it stinks! It is 58.77% C, 13.81% H, and 27.42% N. Its molar mass is
102 g/mol. Determine the molecular formula of cadaverine.

5. A compound which contains one atom of X and two atoms of Y for each three atoms of Z is made
by mixing 5.00 g of X, 1.15×1023 atoms of Y, 0.03 moles of Z atoms. Given that only 4.40 g of
compound is formed. Calculate the atomic weight of Y if the atomic weight of X and Z are 60 and
80 a.m.u. respectively.

6. Titanium, which is used to make air plane engines and frames, can be obtained from titanium
tetrachloride, which in turn is obtained from titanium oxide by the following process:
3 TiO2(s) + 4C (s) + 6Cl2 (g)  3TiCl4(g) + 2CO2(g) + 2CO (g)

A vessel contains 4.15 g TiO2, 5.67 g C and; 6.78 g Cl2, suppose the reaction goes to completion
as written, how many gram of TiCl4 can be produced? (Ti = 48)

7. Two substance P4 & O2 are allowed to react completely to form mixture of P4 O6 & P4 O10 leaving
none of the reactants. Using this information calculate the composition of final mixture when
mentioned amount of P4 & O2 are taken.
P4  3O2 
 P4 O6

P4  5O2 
 P4 O10

(i) If 1 mole P4 & 4 mole of O2

(ii) If 3 mole P4 & 11 mole of O2

(iii) If 3 mole P4 & 13 mole of O2

Don’t ever get comfortable when you have the ability to achieve more.
36 MOLE CONCEPT

8. Fill in the blanks in the following table.


Compound Grams Grams Molality Mole Fraction
Compd Water of Compd of Compd
Na 2 CO3 ______ 250 0.0125 ______

CH3 OH 13.5 150 _____ ______

KNO3 _____ 555 _____ 0.0934

9. A solution of specific gravity 1.6 is 67% by weight. What will be the % by weight of the solution of
same acid if it is diluted to specific gravity 1.2?

10. In what ratio should you mix 0.2M NaNO3 and 0.1M Ca(NO3)2 solution so that in resulting
solution, the concentration of negative ion is 50% greater than conc. of positive ion.

11. What total volume, in litre at 600°C and 1 atm, could be formed by the decomposition of 16 gm of
NH4NO3?
Reaction : 2 NH4NO32N2 + O2 + 4H2O(g).

12. Sulphuric acid is produced when sulphur dioxide reacts with oxygen and water in the presence of
a catalyst : 2SO2(g) + O2 (g) + 2 H2O(l)  2 H2SO4 . If 5.6 mol of SO2 reacts with 4.8 mol of
O2 and a large excess of water, what is the maximum number of moles of H2SO4 that can be
obtained?

13. One gram of an alloy of aluminium and magnesium when heated with excess of dil. HCl forms
magnesium chloride, aluminium chloride and hydrogen. The evolved hydrogen collected at 0°C
has a volume of 1.12 litres at 1 atm pressure. Calculate the composition of the alloy.

14. A sample containing only CaCO3 and MgCO3 is ignited to CaO and MgO. The mixture of oxides
produced weight exactly half as much as the original sample. Calculate the percentages of
CaCO3 and MgCO3 in the sample.

15. Determine the percentage composition of a mixture of anhydrous sodium carbonate and sodium
bicarbonate from the following data:
wt. of the mixture taken = 2g
Loss in weight on heating = 0.11 gm.

16. When 4 gm of a mixture of NaHCO3 and NaCl is heated, 0.66 gm CO2 gas is evolved. Determine
the percentage composition of the original mixture.

17. A power company burns approximately 474 tons of coal per day to produce electricity. If the
sulphur content of the coal is 1.30 % by weight, how many tons SO2 are dumped into the
atmosphere each day?

18. Calculate the percent loss in weight after complete decomposition of a pure sample of potassium
chlorate.
KClO3(s) KCl(s) + O2(g)

19. A sample of calcium carbonate is 80% pure, 25 gm of this sample is treated with excess of
HCl.How much volume of CO2 will be obtained at 1 atm & 273 K?

CatalyseR Eduventures (India) Pvt. Ltd.


MOLE CONCEPT 37

20. Cyclohexanol is dehydrated to cyclohexene on heating with conc. H2SO4. If the yield of this
reaction is 75%, how much cyclohexene will be obtained from 100 g of cyclohexanol ?
(Conc. H2SO4)
C6H12OC6H10

21. How many grams of 90% pure Na2SO4 can be produced from 250 gm of 95% pure NaCl ?

22. Sulphur trioxide may be prepared by the following two reactions :


S8 + 8O2(g)  8SO2(g)

2SO2(g) + O2(g)  2SO3(g)


How many grams of SO3 will be produced from 1 mol of S8?

23. 2PbS + 3O2 2PbO + 2SO2


3SO2 + 2HNO3 + 2H2O  3H2SO4 + 2NO
According to the above sequence of reactions, how much H2SO4 will 1146 gm of PbS produce?

24. Two substance P4& O2 are allowed to react completely to form mixture of P4O6& P4O10 leaving
none of the reactants. Using this information calculate the composition of final mixture when
mentioned amount of P4& O2 are taken.
P4 + 3O2 P4O6
P4 + 5O2 P4O10
(i) If 1 mole P4& 4 mole of O2
(ii) If 3 mole P4& 11 mole of O2
(iii) If 3 mole P4& 13 mole of O2

25. P4S3 + 8O2 P4O10 + 3SO2


Calculate minimum mass of P4S3 is required to produce 1 gm of each product.

26. By the reaction of carbon and oxygen, a mixture of CO and CO2 is obtained. What is the
composition by mass of the mixture obtained when 20 grams of O2 reacts with 12 grams of
carbon ?



Don’t ever get comfortable when you have the ability to achieve more.
38 MOLE CONCEPT

EXERCISE # 04 SUBJECTIVE EXERCISE (LEVEL # 02)

1. A complex compound cobalt has : Co = 22.58%, H = 5.79%, N = 32.2%, O = 12.26% and


Cl = 27.17%. When the compound is heated it lost NH3 to the extent of 32.63% of its original
weight. How many molecules of NH3 are present in the complex compound? Derive empirical
formula of the compound (Co = 59).

2. Hexane  C 6H14  & aniline  C6H7N are partially miscible. At 25°C, 0.5 mole of hexane & 0.5 mol
of aniline are shaken together & allowed to settle. Two liquid layers are formed.On analysis, the
layer A rich in aniline has 10 mol% of hexane while the layer B, rich in hexane has 70 mole% of
hexane. What is the weight ratio of layers A & B?

3. Three different brands of liquid chlorine are available in the market for the use in purifying water
of swimming pools. All are sold at the same rate of Rs 10 per litre and all are water solutions.
Brand A contains 10% hypochlorite (ClO), brand B contains 7% available chlorine (Cl) and brand
C contains 14% sodium hypochlorite (NaClO). All percentage are (w/v) ratios. Which of the three
would you buy?

4. A sea water sample has a density of 1.03 g/cm3 and 2.8% NaCl by mass. A saturated solution of
NaCl in water is 5.45 M NaCl. How much water would have to be evaporated from 1.00 × 106 L of
the sea water before NaCl would precipitate?

5. In a solution the concentrations of CaCl2 is 5M & that of MgCl2 is 5m. The specific gravity of
solution is 1.05, calculate the concentration of Cl in the solution in terms of Molarlity.

6. 3.6 g of Mg is burnt in limited supply of oxygen. The residue was treated with 100 mL of H2 SO 4
(35% by mass,1.26 g mL–1 density). When 2.463 L of H2 at 760 mm Hg at 27C was evolved.
After the reaction, H2 SO 4 was found to have a density of 1.05 g mL–1. Assuming no volume
change in H2 SO 4 solution. Find

(i) % by mass of final H2 SO 4


(ii) % by mass of Mg converted to oxide
(iii) mass of oxygen used. (Mg = 24, S= 32)

7. An impure sample of CuSO4. 5H2O (having 40% purity) undergoes following sequence of
reactions in a reaction flask having large amount of KCN
i. CuSO4.5H2O  CuSO4 + 5H2O ....(A)

ii. CuSO4+KCN  Cu(CN)2 + K2SO4 ....(B)

iii. Cu(CN)2 Cu2(CN)2 + (CN)-2 ....(C)

iv. Cu2(CN)2 + KCN  K3[Cu(CN)4] ....(D)

CatalyseR Eduventures (India) Pvt. Ltd.


MOLE CONCEPT 39

If % yield of react. (A)is 100% (B) is 80% (C) is 60% &(D) is 50%. Calculate
a. wt. of impure sample of CuSO4.5H2O required for producing 28.5 gm of complex
compound K3[Cu(CN)4]
b. vol. of (CN)2 gas produced at STP if wt. of impure sample of CuSO4.5H2O as obtained in
'a' is reacted as above.

8. In a determination of P an aqueous solution of NaH2PO4 is treated with a mixture of ammonium


and magnesium ions to precipitate magnesium ammonium phosphate Mg(NH4)PO4. 6H2O. This
is heated and decomposed to magnesium pyrophosphate, Mg2P2O7 which is weighed. A
solution of NaH2PO4 yielded 1.054 g of Mg2P2O7. What weight of NaH2PO4 was present
originally?

9. A sample of mixture of CaCl2 and NaCl weighing 4.22 gm was treated to precipitate all the Ca as
CaCO3 which was then heated and quantitatively converted to 0.959 gm of CaO. Calculate the
percentage of CaCl2 in the mixture.

10. Potassium superoxide, KO2, is utilised in closed system breathing apparatus. Exhaled air
contains CO2 and H2O, both of which are removed and the removal of water generates oxygen
for breathing by the reaction
1. 4KO2(s) + 2H2O(l) 3O2(g) + 4KOH(s)
2. The potassium hydroxide removes CO2 from the apparatus by the reaction :

3. KOH (s) + CO2 (g)  KHCO3(s)


i. What mass of KO2 generates 20 gm of oxygen ?
ii. What mass of CO2 can be removed from the apparatus by 100 gm of KO2?

11. Sodium chlorate, NaClO3, can be prepared by the following series of reactions:

1. 2KMnO4 + 16 HCl  2 KCl + 2 MnCl2 + 8H2O + 5 Cl2

2. 6Cl2 +6 Ca(OH)2 Ca(ClO3)2 + 5 CaCl2 + 6H2O

3. Ca(ClO3)2 + Na2SO4 CaSO4 + 2 NaClO3


4. What mass of NaClO3 can be prepared from 100 ml of concentrated HCl (density 1.18
gm/ml and 36% by mass)? Assume all other substances are present in excess amounts.

12. A mixture of nitrogen and hydrogen. In the ratio of one mole of nitrogen to three moles of
hydrogen, was partially converted into NH3 so that the final product was a mixture of all these
three gases. The mixture was to have a density of 0.497 g per litre at 25°C and 1.00 atm. What
would be the mass of gas in 22.4 litres at 1 atm and 273 K? Calculate the % composition of this
gaseous mixture by volume.

Don’t ever get comfortable when you have the ability to achieve more.
40 MOLE CONCEPT

13. 0.6872 gm of an organic compound gave on complete combustion 1.466 gm of carbon dioxide
and 0.4283 gm of water. A given weight of the compound when heated with nitric acid and silver
nitrate gave an equal weight of silver chloride. 0.3178 gm of the compound gave 26.0cc of
nitrogen at 15°C and 765 mm pressure. Deduce the empirical formula of the compound?

14. A 10 g sample of a mixture of calcium chloride and sodium chloride is treated with Na2CO3 to
precipitate calcium as calcium carbonate. This CaCO3 is heated to convert all the calcium to CaO
and the final mass of CaO is 1.12gm. Calculate % by mass of NaCl in the original mixture.

15. A mixture of Ferric oxide (Fe2O3) and Al is used as a solid rocket fuel which reacts to give Al2O3
and Fe. No other reactants and products are involved. On complete reaction of 1 mole of Fe2O3,
200 units of energy is released.
1. Write a balance reaction representing the above change.
2. What should be the ratio of masses of Fe2O3 and Al taken so that maximum energy per
unit mass of fuel is released.
3. What would be energy released if 16 kg of Fe2O3 reacts with 2.7 kg of Al.

16. The chief ore of Zn is the sulphide, ZnS. The ore is concentrated by froth floatation process and
then heated in air to convert ZnS to ZnO.
1. 2ZnS + 3O2  2ZnO + 2SO2

2. ZnO + H2SO4  ZnSO4 + H2O

3. 2ZnSO4 + 2H2O  2Zn + 2H2SO4 + O2


i. What mass of Zn will be obtained from a sample of ore containing 291 kg of ZnS.
ii. Calculate the volume of O2 produced at 1 atm & 273 K in part (a).

17. 1 gm sample of KClO3 was heated under such conditions that a part of it decomposed according

to the equation 2KClO3  2 KCl + 3O2

and remaining underwent change according to the equation. 4KClO3  3 KClO4 + KCl
If the amount of O2 evolved was 112 ml at 1 atm and 273 K., calculate the % by weight of KClO4
in the residue.

18. In a determination of P an aqueous solution of NaH2PO4 is treated with a mixture of ammonium


and magnesium ions to precipitate magnesium ammonium phosphate Mg(NH4)PO4. 6H2O. This
is heated and decomposed to magnesium pyrophosphate, Mg2P2O7 which is weighed. A
solution of NaH2PO4 yielded 1.054 g of Mg2P2O7. What weight of NaH2PO4 was present
originally?

CatalyseR Eduventures (India) Pvt. Ltd.


MOLE CONCEPT 41

19. For a hypothetical chemical reaction represented by 3A(g)C(g) + D(g), the following
informations are known.
Information
1. At t = 0, only 1 mole of A is present and the gas has V.D. = 60.
2. At t =30 min, the gaseous mixture consist of all three gases and has a vapour density = 75.
3. Molecular Mass of C = 200
Calculate
i. Molecular weight of A and D.
ii. Moles of each specie at t = 30 min.
20. Equal weights of mercury and iodine are allowed to react completely to form a mixture of
mercurous and mercuric iodide leaving none of the reactants. Calculate the ratio by weight of
Hg2I2 and HgI2 formed (Hg = 200 , I = 127)

21. 124 gm of mixture containing NaHCO3, AlCl3& KNO3 requires 500 ml, 8% w/w NaOH solution
[dNaOH = 1.8 gm/ml] for complete neutralisation. On heating same amount of mixture, it shows
loss in weight of 18.6 gm. Calculate % composition of mixture by moles. Weak base formed
doesn't interfere in reaction. Assume KNO3 does not decompose under given conditions.

22. If the yield of chloroform obtainable from acetone and bleaching powder is 75%. What is the
weight of acetone required for producing 30 gm of chloroform?
2CH3COCH3 + 6CaOCl2  Ca(CH3COO)2 + 2CHCl3 + 3CaCl2 + 2Ca(OH)2

23. A sample of impure Cu2O contians 66.67% of Cu. What is the percentage of pure Cu2O in the
sample?
24. Chloride samples are prepared for analysis by using NaCl, KCl and NH4Cl separately or as

mixture. What minimum volume of 5 % by weight AgNO3 solution (sp.gr, 1.04 g ml–1) must be
added to a sample of 0.3 g in order to ensure complete precipitation of chloride in every possible
case?
25. Consider the following set of reactions

If 0.1 moles of silver salt is taken & wt. of residue obtained is 54 gms then what will be the
molecular mass of
 CH — CH CH 
CH3 —   — CH3
| | | 
 Br Br Br n



Don’t ever get comfortable when you have the ability to achieve more.
42 MOLE CONCEPT

EXERCISE # 05 KVPY, OLYMPIADS QUESTIONS

1. The volume of oxygen at STP required to burn 2.4 g of carbon completely is [KVPY 2014]
(A) 1.12 L (B) 8.96 L (C) 2.24 L (D) 4.48 L

2. Complete reaction of 2.0 g of calcium (at. wt. = 40) with excess HCl produces 1.125 L of H 2 gas.
Complete reaction of the same quantity of another metal “M” with excess HCl produces 1.85 L of
H 2 gas under indentical conditions. The equivalent weight of “M” is closest to [KVPY 2014]
(A) 23 (B) 9 (C) 7 (D) 12

3. When 262 g of xenon (atomic mass = 131) reacted completely with 152 g of fluorine (atomic
mass = 19), a mixture of XeF2 : XeF6 is [KVPY 2015]
(A) 1:2 (B) 1:4 (C) 1:1 (D) 1:3

4. When 22.4 L of C 4H8 at STP is burnt completely, 89.6 L of CO2 gas at STP and 72 g of water
are produced. The volume of the oxygen gas at STP consumed in the reaction is closest to
[KVPY 2015]
(A) 89.6 L (B) 112 L (C) 134.4 L (D) 22.4 L

5. If Avogadro’s number is A 0 , the number of sulphur atoms present in 200 mL of 1 NH2 SO 4 is


[KVPY 2015]
(A) A0 / 5 (B) A0 / 2 (C) A 0 / 10 (D) A0

6. The percentage of nitrogen by mass in ammonium sulphate is closest to


(atomic masses H = 1, N = 14, O = 16, S = 32) [KVPY 2015]
(A) 21% (B) 24% (C) 36% (D) 16%

7. One mole of one of the sodium salts listed below, having carbon content close to 14.3%,
produces 1 mole of carbon dioxide upon heating (atomic mass Na = 23, H = 1, C = 12, O = 16).
The salt is [KVPY 2016]
(A) C2H5 COONa (B) NaHCO3 (C) HCOONa (D) CH3 COONa

8. LiOH reacts with CO2 to form Li2 CO3 (atomic mass of Li = 7). The amount of CO2 (in g)
consumed by 1 g of LiOH is closest to [KVPY 2016]
(A) 0.916 (B) 1.832 (C) 0.544 (D) 1.088

9. A 1.85 g sample of an arsenic-containing pesticide was chemically converted to AsO34 (atomic


mass of As = 74.9) and titrated with Pb 2  to form Pb3  AsO 4 2 . If 20 mL of 0.1 M Pb 2  is
required to reach the equivalence point, the mass percentage of arsenic in the pesticide sample
is closest to [KVPY 2016]
(A) 8.1 (B) 2.3 (C) 5.4 (D) 3.6

CatalyseR Eduventures (India) Pvt. Ltd.


MOLE CONCEPT 43

10. Approximate numbers of moles of hydrogen atoms in 1.006  10 23 molecules of diethyl ether are
[NSEC 2014]
(A) 0.16 (B) 6 (C) 1.67 (D) 3

11. Aluminum carbide  Al4 C3  liberates methane on treatment with water. The grams of aluminum
carbide required to produce 11.2 L of methane under STP conditions is [Given Al = 27]
[NSEC 2014]
(A) 48 (B) 72 (C) 144 (D) 24

12. When 1 L of 0.1 M sulphuric acid solution is allowed to react with 1 L of 0.1 M sodium hydroxide
solution, the amount of sodium sulphate (anhydrous) that can be obtained from the solution
formed and the concentration of H  in the solution respectively are [NSEC 2015]
(A) 3.55 g, 0.1 M (B) 7.10 g, 0.025 M
(C) 3.55 g, 0.025 M (D) 7.10 g, 0.05 M



Don’t ever get comfortable when you have the ability to achieve more.
44 MOLE CONCEPT

EXERCISE # 06 JEE (MAIN) CORNER

1. How many moles of magnesium phosphate, Mg3 PO 4  2 will contain 0.25 mole of oxygen atoms?
[2006]
2 2
(A) 1.25  10 (B) 2.5  10 (C) 0.02 (D) 3.125  10 2

2. Density of a 2.05 M solution of acetic acid in water is 1.02 g/mL. The molarity of the solution is
[2006]
(A) 2.28molkg1 (B) 0.44molkg1 (C) 1.14molkg1 (D) 3.28molkg1

3. The density  in gmL 


1
of a 3.60 M sulphuric acid solution that is 29% H2 SO 4

 molar mass  98 gmol  1


by mass will be [2007]
(A) 1.45 (B) 1.64 (C) 1.88 (D) 1.22
4. In the reaction, [2007]
3 1
2Al  s   6HCl(aq)  2Al  aq   6Cl  aq   3H2  g 
(A) 11.2LH2  g  at STP is produced for every mole HCl (aq) consumed

(B) 6 L HCl(aq) is consumed for every 3LH2  g  produced

(C) 33.6LH2  g  is produced regardless of temperature and pressure for every mole Al that
reacts
(D) 67.2H2  g  at STP is produced for every mole Al that reacts.

5. 29.5 mg of an organic compound containing nitrogen was digested according to Kjeldahl’s


method and the evolved ammonia was absorbed in 20 mL of 0.1 M HCl solution. The excess of
the acid required 15 mL of 0.1 M NaOH solution for complete neutralization. The percentage of
nitrogen in the compound is : [2010]
(A) 59.0 (B) 47.4 (C) 23.7 (D) 29.5

6. A 5.2 molal aqueous solution of methyl alcohol CH3 OH is supplied. What is the mole fraction of
methyl alcohol in the solution ? [2011]
(A) 0.05 (B) 0.1 (C) 0.190 (D) 0.086

7. The density of a solution prepared by desolving 120 g of Urea (mol. mass = 60 u) in 1000 g of
water is 1.15 g/ml. The molarity of this solution is [2012]
(A) 0.5 M (B) 1.78 M (C) 1.02 M (D) 2.05 M

8. A gaseous hydrocarbon gives upon combustion 0.72 g of water 3.08 g of CO 2 . The empirical
formula of hydrocarbon is [2013]
(A) C 3 H4 (B) C6H5 (C) C7H8 (D) C 2 H4

CatalyseR Eduventures (India) Pvt. Ltd.


MOLE CONCEPT 45

9. A compound with molecular mass 180 is acylated with CH3 COCl to get a compound with
molecular mass 390. The number of amino groups present per molecule of the former compound
is: [2013]
(A) 5 (B) 4 (C) 6 (D) 2

10. The molarity of a solution obtained by mixing 750 ml of 0.5 M HCl with 250 ml 2 M HCl will be [2013]
(A) 1M (B) 1.75 M (C) 0.975 M (D) 0.875 M

11. The ratio of masses of Oxygen and Nitrogen in a particular gaseous mixture is 1 : 4. The ratio of
number of their molecules is [2014]
(A) 1 : 4 (B) 7 : 32 (C) 1 : 8 (D) 3 : 16

12. For the estimation of nitrogen, 1.4 g of organic compound was digested by Kjeldahl method and
M
the evolved ammonia was absorbed in 60 mL of sulphuric acid. The unreached acid required
10
M
20 ml of sodium hydroxide for complete neutralization. The percentage of nitrogen in the
10
compound is : [2014]
(A) 3% (B) 5% (C) 6% (D) 10%

13. The molecular formula of a commercial resin used for exchanging ions in water softening is
C8H7 SO3Na (Mole Wt. 206). What would be the maximum uptake of Ca2  ions by the resin when
expressed in mole per gram resin? [2015]
1 2 1 1
(A) (B) (C) (D)
206 309 412 103

14. At 300 K and 1 atm, 15 mL of a gaseous hydrocarbon requires 375 mL air containing 20% O2 by
volume for complete combustion. After combustion the gases occupy 330 mL. Assuming that the
water formed is in liquid form and the volumes were measured at the same temperature and
pressure, the formula of the hydrocarbon is: [2016]
(A) C3H8 (B) C 4H8 (C) C 4H10 (D) C3H6

15. 1 gram of a carbonate  M 2CO3  on treatment with excess HCl produces 0.01186 mole of

CO2 . The molar mass of M 2CO3 in g mol 1 is : [2017]


(A) 84.3 (B) 118.6 (C) 11.86 (D) 1186

16. The most abundant elements by mass in the body of a healthy human adult are:
Oxygen (61.4%); Carbon (22.9%); Hydrogen (10.0%); and Nitrigen (2.6%).
1 2
The weight which a 75 kg person would gain if all H atoms are replaced by H atoms is:
[2017]
(A) 37.5 kg (B) 7.5 kg (C) 10 kg (D) 15 kg


Don’t ever get comfortable when you have the ability to achieve more.
46 MOLE CONCEPT

EXERCISE # 07 JEE (ADVANCED) CORNER

54 57
1. Given that the abundances of isotopes Fe, 56 Fe and Fe are 5%, 90% and 5% respectively,
the atomic mass of Fe is : [2009]
(A) 55.85
(B) 55.95
(C) 55.75
(D) 56.05

2. A student performs a titration with different burettes and finds titre values of 25.0 mL, 25.25 mL
and 25.0 mL. The number of significant figures in the average titre value is : [2010]

1 3
3. Silver (atomic weight  108g mol ) has a density of 10.5 g cm . The number of silver atoms
12
on a surface of area 10 m 2 can be expressed in scientific notation as y  10 x. The value of x
is: [2010]

4. Dissolving 120g of urea (mol. wt. 60) in 1000 g of water gave a solution of density 1.15 g/mL. The
molarity of the solution is- [2011]
(A) 1.78 M
(B) 2.00 M
(C) 2.05 M
(D) 2.22 M

5. A decapeptide (Mol. Wt. 76) on complete hydrolysis gives glycine (Mol. Wt. 75), alanine and
phenylalanine. Glycine contributes 47.0% to the total weight of the hydrolysed products. The
number of glycine units present in the decapeptide is- [2011]

6. The volume (in mL) of 0.1 M AgNO3 required for complete precipitation of chloride ions present

in 30 mL of 0.01 M solution of Cr  H 2O 5 Cl  Cl2 , as silver chloride is close to [2011]

1
7. 29.2% (w/w) HCl stock solution has density of 1.25 gmL . The molecular weight of HCl is 36.5 g

mol 1. The volume (mL) of stock solution required to prepare a 200 mL solution of 0.4 M HCl is:
[2012]

CatalyseR Eduventures (India) Pvt. Ltd.


MOLE CONCEPT 47

COMPREHENSION

1 1
X and Y are two volatile liquids with molar weights of 10 g mol and 40 g mol respectively. Two
cotton plugs, one soaked in X and the other soaked in Y, are simultaneously placed at the ends of a tube
of length L = 24 cm, as shown in the figure. The tube is filled with an inert gas at 1 atmosphere pressure
and a temperature of 300 K. Vapours of X and Y react to form a product which is first observed at a
distance d cm from the plug soaked in X. Take X and Y to have equal molecular diameters and assume
ideal behaviour for the inert gas and the two vapours.

8. The value of d in cm (shown in the figure), as estimated from Graham’s law, is [2014]
(A) 8 (B) 12 (C) 16 (D) 20

9. The experimental value of d is found to be smaller than the estimate obtained using Graham’s
law. This is due to : [2014]
(A) larger mean free path for X as compared to that of Y.
(B) larger mean free path for Y as compared to that of X.
(C) increased collision frequency of Y with the inert gas as compared to that of X with the inert
gas.
(D) increased collision frequency of X with the inert gas as compared to that of Y with the inert
gas.

10. A compound H 2 X with molar weight of 80 g is dissolved in a solvent having density of

0.4 gml 1. Assuming no change in volume upon dissolution, the molarity of a 3.2 molar solution
is [2014]

11. The mole fraction of a solute in a solution is 0.1. At 298 K, molarity of this solution is the same as
its molality. Density of this solution is the same as its molality. Density of this solution at 298 K is

3  MWsolute 
2.0 g cm . The ratio of the molecular weights of the solute and solvent,   , is
 MWsolvent 
[2016]


Don’t ever get comfortable when you have the ability to achieve more.
48 MOLE CONCEPT

ANSWERKEY
ANSWER KEY EXERCISE # 01OBJECTIVE EXERCISE

ANSWER KEY
Q 1 2 3 4 5 6 7 8 9 10

ANS B C A D C A D B B D

Q 11 12 13 14 15 16 17 18 19 20

ANS C C C B C C C A C B

Q 21 22 23 24 25 26 27 28 29 30

ANS B C A A B A B B D D

Q 31 32 33 34 35 36 37 38 39 40

ANS D D A A A A B B B A

Q 41 42 43 44 45 46 47 48 49 50

ANS A C C C A B C A B C

ANSWER KEY EXERCISE # 02 ADVANCED LEVEL EXERCISE

Qs. 1 2 3 4 5 6 7 8 9 10
Ans. ABCD AB ABD ABCD BCD ABCD CD ABCD ABD BD
Qs. 11 12 13 14 15 16 17 18 19 20

Ans. ABC CD ACD AD ABC AB A A B D

Qs. 21 22 23 24 25 26 27 28 29 30
Ans. B C A A B A B B A A
Qs. 31 32 33 34 35 36 37 38
Ans. A C A B A C A A
Qs. 39 40

Ans. A  Q, B  P , C  R , D  S A  Q, B  P , C  R , D  S
Qs. 41 42

Ans. A  R, B  Q, C  P A  R, B  P, C  Q

Qs. 43 44 45 46 47 48 49 50
Ans. 9 6 8 8 2 5 2 5

CatalyseR Eduventures (India) Pvt. Ltd.


MOLE CONCEPT 49

ANSWER KEY EXERCISE # 03 SUBJECTIVE EXERCISE (LEVEL # 01)

1. 7.09  10 7 2. 19.09  106 years


3. (a) 3.41  10 4 mole, (b) 5.68  10 3 mole, (c) 2.05  10 22 atoms

4. C5 H14 N 2 5. 70 6. 9.063 gm
7. (i) 0.5, 0.5; (ii) 2, 1 (iii) 1, 2

8. 0.331, 2.25 104 , 2.81, 0.0482, 321, 5.72


9. 1.288 gm/ml 10. 1:2 11. 50.14 litre 12. 5.6
13. Al=60%, Mg=40% 14. CaCO3=28.4%, MgCO3=71.6%
15. NaHCO3= 14.9%, Na2CO3=85.1% 16. 63%, 37%
17. 12.3 18. 39.18 19. 4.48 litre 20. 61.5 gm
21. 320.3 gm 22. 640.0 23. 470.4 gm 24. 1.1458
25. 21: 11 26. 1.14

ANSWER KEY EXERCISE # 04 ADVANCEDSUBJECTIVE EXERCISE (LEVEL # 02)

1. 5 molecule,  Co  NH 3 5  NO2   Cl2 2. WA : WB  0.524

3. Brand B 4. 9.095  105 lit


5.  Cl    13.36 M 6. (i) 28%, (ii) 33.33%, (iii) 0.8 g

7. (i) 521.25 gm, (ii) 2.24 l


8. (i) 0.5,0.5, (ii)2,1, (iii)1,2
9. 45% 10. (a) 59.17 (b) 61.97 11. 12.9 gm
12. 12.15 gm, N2=14.28%, H2=42.86%, NH3= 42.86%
13. C7H10NCl 14. NaCl =77.8%

15. (i) Fe2O3 + 2Al  Al2O3 + 2Fe, (ii) 80:27, (iii) 10,000 units
16. (a) 117 kg, (b) 20.16 x 103lit
17. 59.72% 18. 1.14 gm.
19. (a) mA= 120, mD=160 (b) nA=2/5, nC=1/5, nD=1/5
20. AlCl3 =33.33, NaHCO3 =50, KNO3 =16.67
21. 9.4 gm 22. 75% 23. 0.532 : 1.00
24. 18.38 ml 25. 495

Don’t ever get comfortable when you have the ability to achieve more.
50 MOLE CONCEPT

ANSWER KEY EXERCISE # 05 KVPY, OLYMPIADS QUESTIONS

Qs. 1 2 3 4 5 6 7 8 9 10
Ans. B D C C C A B A C C
Qs. 11 12
Ans. D D

ANSWER KEY EXERCISE # 06 JEE (MAIN) CORNER

Qs. 1 2 3 4 5 6 7 8 9 10
Ans. B C D D C D D C A D
Qs. 11 12 13 14 15 16
Ans. B D C A A B

ANSWER KEY EXERCISE # 07 JEE (ADVANCED) CORNER

Qs. 1 2 3 4 5 6 7 8 9 10
Ans. B 3 7 C 6 6 8 C D 8
Qs. 11
Ans. 9

CatalyseR Eduventures (India) Pvt. Ltd.


ELECTROSTATICS 1

REDOX REACTION
INDEX

 CONCEPTS IN BRIEF (REDOX REACTION) 51 – 65


 SOLVED EXAMPLES 66 – 72
 EXERCISE # 01 OBJECTIVE EXERCISE 73 – 76
 EXERCISE # 02 ADVANCED EXERCISE 77 – 84
 EXERCISE # 03 SUBJECTIVE EXERCISE (LEVEL # 01) 85 – 87
 EXERCISE # 04 SUBJECTIVE EXERCISE (LEVEL # 02) 88 – 90
 EXERCISE # 05 KVPY, OLYMPIADS QUESTIONS 91
 EXERCISE # 06 JEE (MAIN) CORNER 92
 EXERCISE # 07 JEE (ADVANCED) CORNER 93 – 95
 ANSWER KEYS 96 – 97

WEIGHTAGE OF ‘REDOX REACTION’ IN JEE (MAIN & ADVANCED) in Last Three Years

JEE (MAIN) Formely known as AIEEE

MARK /
YEAR No. Of Qs.
CHEMISTRY TOTAL MARKS
2015 0 0/120
2016 0 0/120
2017 0 0/120

JEE (ADVANCED)

MARK /
YEAR No. Of Qs.
CHEMISTRY TOTAL MARKS
2015 0 0/168
2016 1 3/124
2017 0 0/122

Nothing is impossible, the word itself says ‘I’m possible’!


REDOX REACTION 51

REDOX REACTION
OXIDATION NUMBER OR OXIDATION STATE

Oxidation state of an element in a particular species (atoms, molecules or ions) is the number of
electrons gained or lost by that element during its change from free state into that species. For
example, the oxidation state of Na in NaCl is +1, of calcium in Ca3  PO4 2 is +2, of chlorine in
Cl2 is zero.
(1) Oxidation no. is given positive sign if electrons are lost. oxidation no. is given negative sign if
electrons are gained.

RULES FOR CLACULATION OF OXIDATION NUMBER

Following rules have been arbitrarily adopted to decide oxidation no. of elements on the basis of
their periodic properties.
1. In uncombined state or free state, oxidation number of an element is zero.
2. In combined state oxidation no. of
(a) F is always 1 .
(b) O is 2 . In peroxides it is 1 , in superoxides it is –1/2. However in F2O it is +2.
(c) 1 (i.e., IA, IIA and IIIA metals).
H is +1. In ionic hydrides it is
(d) halogens as halide is always 1 .
(e) sulphur as sulphide is always 2 .
(f) metals is always +ve.
(g) alkali metals (i.e., I A group — Li, Na, K, Rb, Cs, Fr) is always +1.
(h) alkaline earth metals (i.e., II A group—Be, Mg, Ca, Sr, Ba, Ra) is always 2 .
3. The algebraic sum of the oxidation no. of all the atoms in a compound is equal to zero,
e.g. KMnO4 .
Oxidation no. of K + Ox. no. of Mn + (Ox. no. of O)  4 = 0
 1   7   4   2   0
4. The algebraic sum of all the oxidation no. of elements in a radical is equal to the net
2
charge on the radical, e.g., CO3 .

Oxidation no. of C + 3  (Oxidation no. of O) = 2


 4  3   2   2
5. Oxidation number can be zero, +ve, –ve (integer or fraction)
6. Maximum oxidation no. of an element is = Group number (Except O and F)
Minimum oxidation no. of an element is = Group number –8 (Except metals)

Don’t ever get comfortable when you have the ability to achieve more.
52 REDOX REACTION

REDOX REACTION
Oxidation : It is a process in which electrons are lost by an atom, ion or molecule.

e.g, Mg  Mg2+ + 2e–


 Fe2+  Fe3+ + e–

2Cl–  Cl2 + 2e–
 H2O2  O2 + 2H+ + 2e–

Reduction : Reduction is a process in which electrons are gained by an atom, ion or molecule.

Na+ + e– 
 Na I2 + 2e–  2I–

2H2O + 2e–  H2 + 2 OH–

Redox Reactions : Those reactions in which electrons are transferred from one substance to
another are called Redox Reactions.

Zn + Fe2+  Zn2+ + Fe
 MnO2 + 4HCl 
 MnCl2 + Cl2 + 2H2O
Types of Redox Reactions :
1. Combination reactions : A combination reaction may be denoted in the manner:

A+B 
C
Either A and B or both A and B must be in the elemental form for such a reaction to be a redox
reaction. All combustion reactions, which make use of elemental dioxygen, as well as other
reactions involving elements other than dioxygen, are redox reactions. Some important examples
of this category are:
0 0 4 2

C(s)  O2 (g)   CO2 (g)
0 0 2 3

3Mg(s)  N2 (g)   Mg3 N2 (s)

2. Decomposition reactions : Decomposition reactions are the opposite of combination reactions.


Precisely, a decomposition reaction leads to the breakdown of a compound into two or more
components at least one of which must be in the elemental state. Examples of this class of
reactions are:
1  2 0 0
2H2 O( ) 

 2H2 (g)  O 2 (g)
1 1 0 0

2NaH(s)   2Na(s)  H2 (g)

3. Displacement reactions :

(a) Metal displacement: A metal in a compound can be displaced by another metal in the
uncombined state. Metal displacement reactions find many applications in metallurgical
processes in which pure metals are obtained from their compounds in ores.
2  6  2 0 0 2  6  2
CuSO 4 (aq)  Zn(s)  Cu(s)  ZnSO 4 (aq)

CatalyseR Eduventures (India) Pvt. Ltd.


REDOX REACTION 53

(b) Non-metal displacement:

The non-metal displacement redox reactions include hydrogen displacement and a rarely
occurring reaction involving oxygen displacement.
All alkali metals and some alkaline earth metals (Ca, Sr, and Ba) which are very good reductants,
will displace hydrogen from cold water.
0 1  2  2  2 1 0
Ca(s)  2H2 O( )  Ca(OH)2 (aq)  H2 (g)

4. Disproportionation reactions :

Disproportionation reactions are a special type of redox reactions. In a disproportionation reaction


an element in one oxidation state is simultaneously oxidised and reduced. One of the reacting
substances in a disproportionation reaction always contains an element that can exist in at least
three oxidation states.
 1 1 1  2 0
2H2 O 2 (aq)  2H2 O ( )  O 2 (g)
0 3 1
P4 (s)  3OH (aq)  3H2O( ) 
 PH3 (g)  3H2PO 2 (aq)

5. Comproportionation reactions : Reverse of disproportionation is called comproportionation

For Ex. I– + IO3– + H+  I2 + H2O.

Oxidising Agents and Reducing Agents [oxidants and Reductants]

Those species (atoms, molecules and ions) which have tendency to accept the electrons are
known as oxidizing agents or oxidants, whereas those species (atoms, molecules or ions) which
releases the electrons are called reducing agents or reductants. In other words, oxidizing agents
are reduced and reducing agents are oxidized.

(1) Oxidants are substances which :

(a) oxidize other


(b) are reduced themselves
(c) show electronation.
(d) show a decrease in oxidation number during a redox change
(e) has higher oxidation no. in a conjugate pair of redox.

(2) Reductant are substances which :

(a) reduce other


(b) are oxidized themselves.
(c) show de-electronation
(d) show an increase in oxidation no. during a redox change
(e) has lower oxidation no. in a conjugate pair of redox.

Don’t ever get comfortable when you have the ability to achieve more.
54 REDOX REACTION

(3) A redox change is one in which a reductant is oxidized to liberate electrons, which are
then used up by an oxidant to get itself reduced.
M1   M1+n + ne oxidation

M2+n + ne   M2 reduction

M1 + M2+n  M1+n + M2 Redox reaction

Common Oxidising Agents


Cr2O72–  Cr3+
v.f. = 2(6 – 3) = 6
MnO4¯  Mn2+ (acid)
v.f. = 1(7 – 2) = 5
MnO4¯  MnO2 (basic/neutral)
v.f. = 1(7 – 4) = 3
MnO4¯  MnO42– (highly basic)
v.f. = 1(7 – 6) = 1

Common reducing agents and their valency factor :


Fe2+  Fe3+ ; NO2¯   NO3¯
v.f. = 1(3 – 2) = 1 v.f. = 1(5 – 3) = 2
Sn2+  Sn4+ ; H2O2   O2
v.f. = 1(4 – 2) = 2 v.f. = 2(0 – (–1) = 2.

CALCULATION OF n-FACTOR
For calculating the n-factor of any reactant in any reaction. One must know the kind of reaction it
is, the reactions may be classified into the following three types .
(a) Acid-Base Reactions/Neutralization reactions
(b) Redox Reactions
(c) Precipitation Reactions/Double decomposition reactions

(a) Acid-Base Reaction:


According, to the Arrhenius, an acid is a substance that furnishes H+ ion(s) in solution, a base is
a substance that furnishes OH– ion(s) in solution and neutralization is a reaction in which H+ ion
furnished by acid combines with OH– ions furnished by base. The number of H+ ion(s) furnished
per molecule of the acid is its n-factor also called basicity. Similarly the number of OH– ion(s)
furnished by the base per molecule is its n-factor, also called acidity.

CatalyseR Eduventures (India) Pvt. Ltd.


REDOX REACTION 55

Some Examples
H  Cl
HCl  ;  H  HSO4
H2 SO4 
 n  1 n  1
 2H  SO24  ;
H2 SO4   H  H2PO4
H3 SO4 
 n  2 n  1
 3H  PO34  ;
H3PO4  H  H2PO3
H3 SO3 
 n  3 n  1
 2H  HPO4
H3PO3 
 n  2
The n-factor of H3PO3 cannot be 3 as it has only two dissociable H+ ions.
So, its n-factor or dissociable protons is 1 or 2 as one of the H-atoms is
linked with P atom directly.

 CH3 COO  H
CH3 COOH 
Similarly,
 n  1
n-factor of CH3 COOH is 1, because it contains only one dissociable H ion.
Now, we will consider the n-factor of some bases.

Similarly, n-factor of Al  OH3 can also be 1 or 2 or 3, depending upon the number of OH
released.

(b) REDOX REACTIONS

Those reactions which involve the exchange of electrons are called redox reactions. For the
calculation of n-factor of oxidising agent or reducing agent, the method depends upon the change
in oxidation state of the species considered. We will discuss them one by one.
(i) When only one atom undergoing either reduction or oxidation e.g.

Don’t ever get comfortable when you have the ability to achieve more.
56 REDOX REACTION

In such a case, we consider the change in oxidation state of atom undergoing oxidation
or reduction change per molecule as the n-factor of the species.
n-factor   2   1   7   1  5

(ii) Salts which reacts in such a way that only one atom undergoes change in oxidation state
but appears in two products with the same oxidation state :
In such a case the method of calculation of n-factor remains the same i.e., we will
calculate the change in oxidation state of the atom per mole of that substance (reactant).

In this example, oxidation state of Cr changes from +6 to +3 in both the products. So


n-factor = |(+6) × 2 – (+3) × 2| = 6

(iii) Salts which react in such a way that only one atom undergoes change in oxidation state
but goes in two products with different oxidation state as a result of either only oxidation
or only reduction.

In such a case, it is impossible to calculate the n-factor until and unless one
knows that how much of MnO 4 is changing to Mn2 and how much to Mn6  and if one
knows the balanced equation then there is no need of calculation of n-factor.
Nevertheless in such case the n-factor can be calculated by deducing the total change in
oxidation state divided by total number of atom undergoing reduction/oxidation change.
So, for the calculation of n-factor in the above example, out of three moles of MnO 4 , two
moles are being converted to Mn2 and one mole changes to Mn6  So total decrease in
oxidation state of Mn.
= | [2 × (+2) – 2 × (+7)] | + | [1 × (+6) – 1 × (+7)]|
= | 4 – 14 | + | 6 – 7 | = 11
So, n-factor  11
3

CatalyseR Eduventures (India) Pvt. Ltd.


REDOX REACTION 57

(iv) Salts which react in such a way that only one atom undergoes change in oxidation state
in two product, in one product with changed oxidation state and in other product with
same oxidation state as that of reactant.
In such case also one cannot calculate the n-factor without knowing the balanced
chemical equation because one must know how much of atom has changed its oxidation
state. For example.
K2Cr2O7 + 14HCl  2KCl + 2CrCl3 + 3Cl2 + 7H2O

Let us calculate the n-factor of HCl. Out of 14 moles of Cl– (in HCl) only 6 moles of Cl–
are changing its oxidation state from –1 to 0 in the product Cl2 and the oxidation state of

remaining 8 Cl– ions remains same in KCl and CrCl3. So, total no. of moles of electrons
lost by 14 moles of HCl is 6. So each mole of HCl takes up 6/14 i.e., 3/7 moles of
electrons and hence n-factor of HCl is 3/7.

(v) Salts which react in such a way that two or more atoms in the salt undergoes change in
oxidation states as a result of either oxidation or reduction. Let us consider the following
example,
FeC2O4   Fe3+ + 2CO2

In this case, the oxidation of both Fe2+ and C3+ are changing from + 2 and +3 to +3 and
+4 respectively. In such a case we will calculate the n-factor of the salt as the total
increase or decrease in oxidation state per mole of the salt. As one can see that one
mole of FeC2O4 contains one mole of Fe2+ and one mole of C2O42– (i.e. 2 carbon

atoms per mole of C2O42–.


Total change in oxidation state
= | 1 × (+2) – 1 × (+3) | + | 2 × (+3) – 2 × (+4) | = 1 + 2 = 3
So, n-factor of FeC2O4 is 3
(vi) Salts which react in such a way that two atoms undergoing change in oxidation state
but one undergoing oxidation and other reduction reaction. In such a case one has to
calculate the change in oxidation state of either the atom being oxidized or the atom
being reduced. For example.

In this reaction, the oxidation state of N is increasing by 6 units and that of Cr is


decreasing by 6 unit. So, we can consider either oxidation or reduction product for the
calculation of n- factor and it will be the same.
n-factor of (NH4)2Cr2O7 considering oxidation = |(-3) ×2 – (0) × 2| = 6
n-factor of (NH4)2Cr2O7 considering reduction
= |(+6) × 2 – (+3) × 2| = 6

Don’t ever get comfortable when you have the ability to achieve more.
58 REDOX REACTION

(vii) Species which undergoes disproportionation reaction: Those reaction in which oxidant
and reductant are the same species or the same element from the species is getting
oxidized as well as reduced.
When the number of moles of atoms being oxidized is equal to the number of moles of
atoms being reduced.
The n-factor can be calculated by knowing the balanced chemical equation and
considering any of the change taking place. Say for example,
2H2O2   2H2O + O2
Out of 2 moles of H2O2 consumed in the reaction, one mole of H2O2 is being oxidized
(H2O2   O2) and one mole of H2O2 is being reduced (H2O2  2H2O).
First consider the oxidation reaction

n-factor = |2 × 0 – (–1) × 2| = 2 Again, considering reduction reaction

n-factor = |(-2) × 2 – (-1) × 2| = 2


So, n-factor of H2O2 either considering oxidation or reduction reaction is same i.e. 2.

(c) Precipitation/Double Decomposition Reactions :


In such reaction, there is no change in oxidation state of any atom. The n-factor of the salt can
be calculated by multiplying the oxidation state of the cation/anion by total no. of atoms per
molecule of the salt.
For example

for BaCl2
n-factor  Oxidation state of Ba atom in BaCl2  number of Ba atoms in 1 molecule of
BaCl2   2  1  2
for Na 2 SO 4

n-factor  Oxidation state of Na  number of Na-atoms in 1 molecule of Na2SO4   1  2  2.

CatalyseR Eduventures (India) Pvt. Ltd.


REDOX REACTION 59

BALANCING OF REDOX EQUATIONS

(A) Ion electron method


It involves three sets of rules depending upon the nature of medium (i.e. neutral, acid or alkaline)
in which reaction occurs.

(a) Neutral medium :


e.g., H2C2O4 + KMnO4   CO2 + K2O + MnO4 + H2O
Step 1. Select the oxidatn, reductant atoms and write their half reactions, one representing
oxidation and other reduction.
C2+3   2C+4 + 2e

5e + Mn+7   Mn+2
Step 2. Balance the no. of electrons and add the two equations.
5C2+3   10C+4 + 10e

10e + 2Mn+7  2Mn+2

5C2+3 + 2Mn+7   10C+4 + 2Mn+2


Step 3. Write complete molecule of the reductant and oxidant from which respective redox
atoms were obtained
5H2C2O4 + 2KMnO4   10CO2 + 2MnO
Step 4. Balance other atoms if any (except H and O)
In above example K is unbalanced, therefore,
5H2C2O4 + 2KMnO4  10CO2 + 2MnO + K2O
(Mentioned as product)

Step 5. Balance O atom using H2O on desired side.


5H2C2O4 + 2KMnO4  10CO2 + 2MnO + K2O + 5H2O

(b) Acidic medium :


e.g., NO3– + H2S 
H
HSO4– + NH4+
Produced like neutral medium for step 1 to 4.
Step 1. 8e + N+5   N–3
S–2  S+6 + 8e

Step 2. N+5 + S–2  N–3 + S+6


Step 3. NO3– + H2S   NH4+ + HSO4–
Step 4. No other atom (except H and O) is unbalanced and thus, no need for this step.

Don’t ever get comfortable when you have the ability to achieve more.
60 REDOX REACTION

Step 5. Balance O atom : Balancing of O atom is made by using H2O and H+ ions.

Add desired molecules of H2O on the side deficient with O atom and double H+ on
opposite side. Therefore,
H2O + NO3– + H2S 
H
NH4+ + HSO4– + 2H+

Step 6. Balance charge by H+


3H+ + H2O + NO3– + H2S 
H
NH4+ + HSO4– + 2H+
 Finally balanced equation is ,
H+ + H2O + NO3– + H2S 
H
NH4+ + HSO4–

(c) Alkaline medium :


OH¯
e.g., Fe + N2H4   Fe(OH)2 + NH3

Proceed like neutral medium for step 1 to 4

Step 1. Fe  Fe+2 + 2e


2e + N2–2  2N–3

Step 2. Fe + N2–2  Fe+2 + 2N–3

Step 3. Fe + N2H4  Fe(OH)2 +2NH3

Step 4. No other atom (except H and O) is unbalanced and thus, no need for this step.

Step 5. Balance O atom: Balancing of O atom is made by using H2O and OH– ions.
Add desired molecules of H2O on the side rich with O atoms and

double OH– on opposite side. Therefore,


4OH– + Fe + N2H4  Fe(OH)2 + 2NH3 + 2H2O

Step 6. Balance charge by H+:


4OH– + 4H+ + Fe + N2H4  Fe(OH)2 + 2NH3 + 2H2O

 finally balanced equation is ,


2H2O + Fe + N2H4  Fe(OH)2 + 2NH3

CatalyseR Eduventures (India) Pvt. Ltd.


REDOX REACTION 61

BALANCING OF HALF REACTIONS


Example 1. I2   IO3–
Step 1. Balance atoms other than O and H if needed i.e.,
I2   2IO3–

Step 2. Balance O atoms using H+ and H2O as reported earlier.

I2 + 6H2O   2IO3– + 12H+


Step 3. Balance charge by electrons .
I2 + 6H2O   2IO3– + 12H+ + 10e
This is balanced half reaction.
Example 2. S2O3–2   SO2

Step 1. As above S2O3–2   2SO2

Step 2. Balance O atom by H2O and OH– as reported earlier.

2OH– + S2O3–2  2SO2 + H2O

Step 3. Balance charge by electrons.


2OH– + S2O3–2   2SO2 + H2O + 4e
This is balanced half reaction

(B) Oxidation Number Method : In writing equations for oxidation-reduction reactions, just as for
other reactions, the compositions and formulas must be known for the substances that react and
for the products that are formed. The oxidation number method is now best illustrated in the
following steps:

Step 1: Write the correct formula for each reactant and product.

Step 2: Identify atoms which undergo change in oxidation number in the reaction by assigning the
oxidation number to all elements in the reaction.

Step 3: Calculate the increase or decrease in the oxidation number per atom and for the entire
molecule/ion in which it occurs. If these are not equal then multiply by suitable coefficients so that
these become equal. (If you realise that two substances are reduced and nothing is oxidised or
vice-versa, something is wrong. Either the formulas of reactants or products are wrong or the
oxidation numbers have not been assigned properly).

Step 4: Ascertain the involvement of ions if the reaction is taking place in water, add H+ or OH– ions to
the expression on the appropriate side so that the total ionic charges of reactants and products
are equal. If the reaction is carried out in acidic solution, use H+ ions in the equation; if in basic
solution, use OH– ions.

Don’t ever get comfortable when you have the ability to achieve more.
62 REDOX REACTION

Step 5: Make the numbers of hydrogen atoms in the expression on the two sides equal by adding water
(H2O) molecules to the reactants or products. Now, also check the number of oxygen atoms. If
there are the same number of oxygen atoms in reactants and products. the equation then
represents

Ex. Write the net ionic equation for the reaction of potassium dichromate (VI), K2Cr2O7 with sodium
sulphite (Na2SO3) in an acid solution to give chromium(III) ion and the sulphate ion.

Step 1: The skeletal ionic equation is:


Cr2O72–(aq) + SO32–(aq)  Cr3+(aq) + SO42–(aq)

Step 2: Assign oxidation numbers for Cr and S.


6 2 4  2 3 6  2
Cr2 O72  (aq) SO32  (aq) 
 Cr(aq)  SO24 (aq)
This indicates that the dichromate ion is the oxidant and the sulphite ion is the reductant.
Step 3: Calculate the increase and decrease of oxidation number, and make them equal:
6 2 4  2 3 6  2
Cr2 O72  (aq)  3SO32  (aq) 
 2Cr 3  (aq)  3SO24 (aq)

Step 4: As the reaction occurs in the acidic medium, and further the ionic charges are not equal on both
the sides, add 8H+ on the left to make ionic charges equal
Cr2O72–(aq) + 3SO32–(aq) + 8H+(aq)  2Cr3+(aq) + 3SO42–(aq) + 4H2O(l)

Step 5: Finally, count the hydrogen atoms, and add appropriate number of water molecules (i.e., 4H2O)
on the right to achieve balanced redox change.

VOLUMETRIC ANALYSIS
TITRATION

The process of determination of concentration of a solution with the help of a solution of known
concentration (standard solution) is called titration.
Titration is divided into following three categories.
(A) Simple Titration
(i) Acid base titration
(ii) Double Titration
(iii) Back Titration
(B) Redox titration
(C) Iodimetric and Iodometric Titration

CatalyseR Eduventures (India) Pvt. Ltd.


REDOX REACTION 63

(A) Simple Titration :


NAVA (litre) = NB  VB (litre)

Simple titrations can be studied under following three headings :


(i) Acid base titration (ii) Double Titration (iii) Back Titration

(i) ABT (Acid Base Titration) :


In this type of titration an acid is reacted with base
For ABT, Meq. of acid = Meq. of base
NV
1 1  N2V2

(ii) DOUBLE TITRATION :


If an aqueous solution contains a mixture of any two of the three NaOH, NaHCO3 and
Na2CO3 and it has to be titrated against an acid HCl or H2SO4, it will require two indicators to
determine the strength of the bases present. The two indicators used are phenolphthalein and
methyl orange.

1. Titration of the solution containing both NaOH and Na2CO3:


A given volume of the aqueous solution is titrated with an acid of known normality using
phenolphthalein indicator. Suppose ‘a’ milli equivalents of acid are used in the first end point then,
milli equivalent of NaOH + ½ milli equivalent of Na2CO3
= milli equivalent of acid = a …(1)
Now in the same already titrated solution methyl orange indicator is added and again titrated to
the end point. Suppose ‘b’ milli equivalents of the acid are used at the second end point.
½ milli equivalents of Na2CO3 = milli equivalents of acid = b …(2)
From equation (1) and (2)
Milli equivalents of acid used by Na2CO3 = 2b
 milli equivalents of Na2CO3

Milli equivalents of acid used by NaOH = a – b


= milli equivalent of NaOH
Knowing the milli equivalents of Na 2 CO3 or NaOH and the volume of the solution titrated, their
normality can be calculated.
2. Titration of the solution containing both Na 2 CO3 and NaHCO3 .

Given volume of the solution is titrated by an acid using phenolphthalein indicator. Suppose ‘a’
milli equivalents of acid are used in the first end point. Then
1/2 milli equivalent of Na2CO3 = milli equivalents of acid = a …(1)
Now in the same already titrated solution methyl orange indicator is added and again titrated to
the end point. Suppose ‘b’ milli equivalents of the acid are used at the second end point. Then
1/2 milli equivalents of Na2CO3 + milli equivalents of NaHCO3
= milli equivalents of acid = b ..(2)

Don’t ever get comfortable when you have the ability to achieve more.
64 REDOX REACTION

From equation (1) and (2)


Milli equivalents of acid used by Na2CO3 = 2a
 milli equivalents of Na2CO3
Milli equivalents of acid used by NaHCO3 =b–a
= milli equivalent of NaHCO3
Knowing the milli equivalents of the base and volume of the solution titrated, the normality
(strength) of the bases can be calculated.

(iii) BACK TITRATION :

Let us consider that we have an impure solid substance ‘Z’ weighing ‘w’ g and we are required
to calculate the percentage purity of ‘Z’ in the sample. We are also provided with two solutions
‘X’ and ‘Y’, where the concentration of ‘Y’ is known (N1) and that of ‘X’ is unknown. For
the back titration to work, following conditions are to be satisfied

(a) Compounds ‘X’, ‘Y’ and ‘Z’ should be such that ‘X’ and ‘Y’ reacts with each other.
(b) ‘X’ and pure ‘Z’ also reacts with each other but the impurity present in ‘Z’ does not
react with ‘X’.
Z + X (excess)  Product 1
Remaining (X) + Y  Product 2

Note : Product 1 should not react with Y


Milli equivalent of Y  N2 V2
Where N2 and V2 (ml) is the normality and volume of Y
Initial mili equivalent of X  N1V1
Where N1 and V1 (ml) is the normality and volume of X
Remaining milli equivalents of X after reacting with Y  N1V1  N2 V2
Remaining milli equivalents of X = milli equivalents of Z
a  1000
N1V1  N2 V2 
Equivalent weight
where ‘a’ is the weight of pure Z which is reacted.
Molecular Weight N1V1  N2 V2 
a
n  factor
N1V1  N2 V2  Molar Mass of ' Z'
 Percentage purity of ' Z '    100
n  factor W

(B) REDOX TITRATION :


It is a type of simple titration in which a solution of oxidising agent is reacted with a solution of
reducing agent,
For redox titration, Meq. of reducing agent = Meq. of oxidising agent

CatalyseR Eduventures (India) Pvt. Ltd.


REDOX REACTION 65

Standard Solution of oxidising agent


(solution of known conc. N2)

After
V2 ml complete reaction V3 ml

Reducing agent (solution of unknown


V1 ml concentration. N1) V1 ml

In the above figure Meq. of reducing agent  N1V1

Meq. of oxidising agent  N2  V2  V3   N1V1  N2  V2  V3 


Estimation of By titrating with Reactions
1. Fe2+ MnO4¯ Fe2+   Fe3+ + e–
MnO 4  8H  5e   Mn2   4H2 O

2. Fe2+ Cr2O72– Fe2+  Fe3+ + e–

Cr2O72– + 14H+ + 6e–  2Cr3+ + 7H2O

3. C2O42– MnO4¯ C2O42–  2CO2 + 2e–


MnO 4  8H  5e   Mn2   4H2 O
(C) IODIMETRIC AND IODOMETRIC TITRATIONS
The redox-titration using iodine directly or indirectly as an oxidizing agent are called Iodine
Titrations. There are of two types.
(a) Iodimetric Titrations :
Iodimetric titrations are defined as those iodine titrations in which a standard iodine solution is
used as an oxidant and iodine is directly titrated against a reducing agent.
(i) 2Na2S2O3 + I2  Na2S4O6 + 2NaI
(ii) Na2SO3 + I2 + H2O   Na2SO4 + 2HI
(b) Iodometric Titrations :
Iodometric titrations are defined as those iodine titrations in which some oxidizing agent
liberates iodine from an iodine solution and then liberated iodine is titrated with a standard
solution of a reducing agent added from a Burette.
(i) 2CuSO4 +4KI  Cu2I2+ 2K2SO4 + I2
(ii) 2KMnO4 + 3H2SO4  K2SO4 + 2MnSO4 + 3H2O + 5O
10KI + 5H2SO4 + 5O  5K2SO4 + 5H2O + 5I2
(iii)  K2SO4  Cr2  SO4 3  4H2O  3O
K 2Cr2O7  4H2SO4 
6KI  3H2SO 4  3O  3K 2 SO4  3H2O  3I2
In the above reactions, the liberated iodine is titrated with a standard sodium thiosulphate
2Na2 S 2 O3  I2  Na2 S 4 O6  2NaI



Don’t ever get comfortable when you have the ability to achieve more.
66 REDOX REACTION

SOLVED EXAMPLES

Ex. 1 Find the oxidation number of


(a) S in SO24 ion (b) S in HSO3 ion
2 
(c) Pt in PtCl6  (d) Mn in MnO4  ion
Sol. (a) Let the oxidation number of S be x.
We know that Ox. no. of O = –2
So Ox. no S + 4 (Ox. no. O) = –2  x + 4(–2) = –2
or x – 8 = –2  x = + 8 – 2 = + 6
The oxidation number of S in SO24 ion is +6.

(b) Let the oxidation number of S be x in HSO3– ion.


We know that Ox. no. of H = +1
Oxidation number of O = –2
So Ox. no. H + Ox. no. S + 3 (Ox. no. O) = –1
+ 1 + x + 3(–2) = –1  +1 + x – 6 = –1  x – 5 = –1
or x = + 5 – 1 = +4  The oxidation number of S in HSO3 ion is +4.
(c) Let oxidation number of Pt be x. We know that Ox. no. of Cl = –1
So Ox. no. Pt + 6 (Ox. no. Cl) = –2  x + 6(–1) = –2
or x – 6 = –2  The oxidation number of Pt in [Pt(Cl)6]2– ion is +4.
(d) Let oxidation number of Mn be x. We know that Ox. no. of O = –2
So Ox.no. Mn + 4 (Ox. no. O) = –1
 x + 4(–2) = –1  x – 8 = –1 or x = + 8 –1 = +7.
The oxidation number of Mn in [MnO4]– ion is +7.

Ex. 2 Which of the following species, do not show disproportionation reaction and why?
CIO–, CIO2–, CIO3– and ClO4–
Also write reaction for each of the species that disproportionates.

Sol. Among the oxoanions of chlorine listed above, CIO4– does not disproportionate because in this
oxoanion chlorine is present in its highest oxidation state that is, +7. The disproportionation
reactions for the other three oxoanions of chlorine are as follows:
1 1 5
3ClO   2Cl  ClO3
3 5 1
h
6ClO 2  4ClO 3  2Cl
5 1 7
4ClO 3 
 Cl  3ClO 4

CatalyseR Eduventures (India) Pvt. Ltd.


REDOX REACTION 67

Ex. 3 Suggest a scheme of classiflcation of the following redox reactions.


(a) N2(g) + O2(g)  2NO(g)
1
(b) Pb(NO3)2(s)  PbO(s) + 2NO2(g) + O (g)
2 2
(c) NaH(s) + H2O(l)  NaOH(aq) + H2(g)
– – –
(d) 2NO2(g) + 2OH (aq)  NO2 (aq) + NO3 (aq) + H2O(l)

Sol. In reaction (a), the compound nitric oxide is formed by the combination of the elemental
substances, nitrogen and oxygen ; therefore, this is an example of combination redox reaction.
The reaction (b) involves the breaking down of lead nitrate into three components; therefore, this
is categorised under decomposition redox reaction. In reaction (c), hydrogen of water has been
displaced by hydride ion into dihydrogen gas. Therefore, this may be called as displacement

redox reaction. The reaction (d) involves disproportionation of NO2 (+4 state) into NO2 (+3

state) and NO3 (+5 state). Therefore reaction (d) is an example of disproportionation redox
reaction.

Ex. 4 Why do the following reactions proceed differently?


Pb3O4 + 8HCl  3PbCl2 + Cl2 + 4H2O
and Pb3O4 + 4HNO3   2Pb(NO3)2 + PbO2 + 2H2O
Sol. Pb3O4 is actually a stoichiometric mixture of 2 mol of PbO and 1 mol of PbO2. In PbO2, lead is
present in +4 oxidation state. whereas the stable oxidation state of lead in PbO is +2. PbO2 thus

can act as an oxidant (oxidising agent) and, therefore, can oxidise Cl ion of HCI into chlorine.
We may also keep in mind that PbO is a basic oxide. Therefore, the reaction
Pb3O4 + 8HCl   3PbCl2 + Cl2 + 4H2O
can be splitted into two reactions namely:
2PbO + 4HCl  2PbCl2 + 2H2O (acid-base reaction)
4 1 2 0
PbO2  4HCl 
PbCl2  Cl2  2H2 O (redox reaction)
Since HNO3 itself is an oxidising agent therefore, it is unlikely that the reaction may occur
between PbO2 and HNO3. However, the acid-base reaction occurs between PbO and HNO3 as:
2PbO + 4HNO3   2Pb(NO3)2 + 2H2O
It is the passive nature of PbO2 against HNO3 that makes the reaction different from the one that
follows with HCl.

Don’t ever get comfortable when you have the ability to achieve more.
68 REDOX REACTION

Ex.5 Calculate individual oxidation number of each s -atom in Na2S2O3 (sodium thio sulphate) with the
help of its structure.
Sol. Structure :

I (central S-atom) is sp3 hybridised (25% s-character) and  (terminal S-atom) is sp2 hybridised
(33% S-character). Therefore terminal sulphur atom is more electronegative than central sulphur
atom. Now the shared pair electrons are counted with terminal S-atom.

 I , S-atom : No. of electrons in the valence snell = 6


: No. of electrons left affer bonding = 0
Oxidation No. = 6 – 0 = + 6

II , S-atom : No. of electrons in the valence shell = 6


No. of electrons left affrer bonding = 8
Oxidation No. = 6 – 8 = – 2
62
Now you can also calculate Average Oxidation No. = = + 2 (as we have calculated before)
2

Ex.6 Calculate the amount of oxygen required to produce enough carbon mono oxide on reaction with
carbon which can reduce 1.6 kg Fe2O3.
Sol. Method 1 : (Mole concept)
160 328
Fe2O3  3CO 
 2Fe  3CO2
1600 g

3  28
CO required to reduce 1600 g Fe2O3 =  1600 = 3 × 28 × 10 = 840 g
160
1 16 28
C O2  CO
2
16
O2 required to produce 840g CO =  840 = 480 g.
28
Method 2 : (Equivalent concept)
Eq. of O2 = eq. of Fe2O3
0
W 1600 ( 3)
  Fe 2O 3  Fe
8 160 / 6
v.f = 2(3 – 0) = 6

8  1600  6
W= = 480 g
160

CatalyseR Eduventures (India) Pvt. Ltd.


REDOX REACTION 69

Ex.7 Find the number of moles of kMnO4 needed to oxidise one mole Cu2S in acidic medium.

Sol. The reaction is KMnO4 + Cu2S    Mn2+ + Cu2+ + SO2


v.f. = 5 v.f. = 2(2 – 1) + 1 (4 – (–2))

=8

From law of equivalence


equivalents of Cu2S = equivalents of KMnO4
or, moles of Cu2S × v.f = moles of KMnO4 × v.f
or, 1 × 8 = moles of KMnO4 × 5
 moles of KMnO4 = 8/5

Ex.8 25 gm sample of ferrous sulphate (FeSO4. 7H2O) dissolved in water and dilute H2SO4 and made 1
ltr. 25 ml of solution required 10 ml of 0.04 M KMnO4 solution for complete oxidation. Find the
percentage of FeSO4. 7H2O (molecular weight = 278) in the sample
Sol. m eq. of KMnO4 = 10 × 0.04 × 5 = 2 (in 25 ml)
m eq. of KMnO4 = 2 × 40 = 80 (in 1000 ml)
 m eq. of FeSO4 . 7H2O = 80

80  278
% of FeSO4 . 7H2O = × 100 = 88.96
1000  25

Ex.9 5 g of a sample of Ca(OH)2 is dissolved in 50 ml of 0.5 N HCl solution. The excess of HCl is back
titrated with 0.3 N NaOH. The vol of NaOH used is 10 ml. Calculate % purity of Ca(OH)2.
Sol. Let pure Ca(OH)2 = xg

Ca(OH)2 + 2HCl  CaCl2 + 2H2O


(excess)

HCl + NaOH  NaCl + H2O


(left over)
equivalent of Ca(OH)2 = eq. of HCl – eq. of NaOH

x2
= 50 × 0.5 × 10–3 – 10 × 0.3 × 10–3
74
or x = 0.814 g
0.814
% purity   100  16.28%
5

Don’t ever get comfortable when you have the ability to achieve more.
70 REDOX REACTION

Ex.10 40 ml N/10 HCl and 60 ml N/20 KOH are mixed together. Calculate the normality of the acid or
base left. What is the normality of the salt formed in the solution?
1 40
Sol : Milli equivalents of HCl = N × V (ml)  4
10
1 60
Milli equivalents of KOH  N  V (ml)  3
20
One milli equivalent of an acid neutralizes one milli equivalent of a base
Milli equivalent of HCl left =4–3=1
Total volume of the solution = 40 + 60 = 100 ml
Milli equivalents of HCl = N × V (ml)
1 = N × 100
Normality (N) of HCl left in solution = 0.01
Salt formed = Milli equivalent of acid or base neutralized
Milli equivalents of the salt formed = N × V (ml)
3 = N × 100
Normality (N) of salt formed = 0.03

Ex.11 NaOH and Na2CO3 are dissolved in 200 ml aqueous solution. In the presence of phenolphthalein
indicator. 17.5 ml of 0.1 N HCl are used to titrate this solution. Now methyl orange is added in the
same solution titrated and it requires 2.5 ml of the same HCl. Calculate the normality of NaOH
and Na2CO3 and their mass present in the solution.
Sol.: Milli equivalents (a) of HCl used in the presence of phenolphthalein indicator.
= N  V (ml) = 0.1  17.5 = 1.75
1.75 (a) = milli. eq. of NaOH + 1/2 milli eq. of Na2CO3 ...(1)
Milli eq. (b) of HCl used in the presence of methyl orange indicator
= N  V (ml) = 0.1  2.5 = 0.25
0.25 (b) = 1/2 milli equivalents of Na2CO3 …(2)
For Na2CO3 solution: From equation (2)
Milli eq. of acid used by Na2CO3 = 2b = 2  0.25 = 0.5
Volume of Na2CO3 solution = 200 ml
Suppose, Normality of Na2CO3 =N
Milli equivalents of Na2CO3 =N  V (ml) = 200N
Putting equivalents of acid and Na2CO3 equal.
200N = 0.5
1
or (Normality of Na2CO3 solution) N 
400
Mass of Na2CO3 =N  E  V (litre)
1
(E for Na2CO3 = 53)   53  0.2 = 0.0265 gram
400

CatalyseR Eduventures (India) Pvt. Ltd.


REDOX REACTION 71

For NaOH solution: From equation (1) and (2)


Milli eq. acid used by NaOH = a – b = 1.75 – 0.25
= 1.50
Volume of NaOH solution = 200 ml
Suppose, Normality of NaOH solution =N
Mili eq. of NaOH = N  V (ml) = 200 N
Putting the mili eq. of NaOH and acid used equal 200 N = 1.5
1.5
(Normality of NaOH solution) N
200
1.5
Mass of NaOH = N  E  (V litres)   40  0.2
200
(E for NaOH = 40) = 0.06g

Ex.12 Both Na2CO3 and NaHCO3 are present in an aqueous solution. In the presence of
phenolphthalein indicator 10 ml of this solution requires 2.5 ml of 0.1M H2SO4 for titration. After
this methyl orange is added in the same solution and titration requires 5 ml M H2SO4. Calculate
the concentration of Na2CO3 and NaHCO3 in g/litre.

Sol. Milli eq. (a) of H2SO4 used in the presence of phenolphthalein indicator
=N  V (ml) = 0.1  2  2.5 = 0.5
a = 0.5 = 1 milli equivalents of Na2CO3 …(1)
2
Milli. eq. (b) of NaHCO3 + ½ milli eq. of Na2CO3 …(2)
For Na2CO3 solution: From equation (1)
Milli eq. of acid used by Na2CO3 = 2  0.5 = 1
Suppose, Normality of Na2CO3 solution = N Volume of Na2CO3 solution taken = 10 ml
Milli eq. of Na2CO3 taken = N  V (ml) = 10 N
Putting the milli eq. of H2SO4 and Na2CO3 equal, 1 = 10 N or (Normality of Na2CO3) N = 0.1
Strength (S) in g/litre =N  E = 0.1  53 (E for Na2CO3 = 53) = 5.3 g/litre
For NaHCO3 solution: From equations (1) and (2) milli eq. of acid used by
NaHCO3 = b – a = 1.0 – 0.5 = 0.5
Suppose, Normality of NaHCO3 solution =N
Volume of NaHCO3 solution taken = 10 ml
Milli equivalents of NaHCO3 taken = 10 N
Putting the milli eq. of H2SO4 and NaHCO3 equal, 0.5 = 10 N
or (Normality of NaHCO3 solution) N = 0.05
Strength (S) in g/litre = N  E
(E for NaHCO3 = 84) = 0.05  84= 4.2g/litre

Don’t ever get comfortable when you have the ability to achieve more.
72 REDOX REACTION

Ex.12. 0.5 g sample containing MnO2 is treated with HCl, liberating Cl2. The Cl2 is passed into a

solution of KI and 30.0 cm3 of 0.1 M Na2S2O3 are required to titrate the liberated iodine.
Calculate the percentage of MnO2 in sample. (At. Wt. of Mn = 55).
HCl KI Na2 S2 O3
Sol. MnO2   Cl2  I2  NaI  Na2S4O6
Redox change are : 2e  I02 
 2I

 S4
5/2
2S22   2e
2e  Mn4  
 Mn2 

The reactions suggest that ,


Meq. of MnO2 = Meq. of Cl2 formed
= Meq. of I2 liberated
= Meq. of Na2S2O3 used
w
  1000  0.1 1 30
M/ 2
[  NNa2S2O3  MNa2S2O3 since valency factor = 1, see redox changes for Na2S2O3]

or wMnO2  0.1305

0.1305
 Purity of MnO2   100  26.1%
0.5


CatalyseR Eduventures (India) Pvt. Ltd.


REDOX REACTION 73

EXERCISE # 01 OBJECTIVE EXERCISE

1. The number of moles of KMnO 4 that will be needed to react completely with one mole of ferrous
oxalate in acid solution is
(A) 3/5 (B) 2/5 (C) 4/5 (D) 1
2. The number of moles of KMnO 4 that will be needed to react with one mole of sulphite ions in
acidic solution is
(A) 2/5 (B) 3/5 (C) 4/5 (D) 1

3. The normality of 0.3 M phosphorus acid H3PO3  is -


(A) 0.1 (B) 0.9 (C) 0.3 (D) 0.6
4. One mole of calcium phosphide on reaction with excess of water gives
(A) one mole of phosphine (B) Two moles of phsophoric acid
(C) Two moles of phosphine (D) One mole of phosphorus pentoxide
5. An aqueous solution of 6.3 gm of oxalic acid dihydrate is made upto 250 ml. The volume of 0.1 N
NaOH required to completely neutralize 10 ml of this solution is
(A) 40 ml (B) 20 ml (C) 10 ml (D) 4 ml
6. In the standarization of Na 2S 2 O3 using K 2 Cr2O7 by iodometry the equivalent mass of K 2 Cr2O7 is
M. Mass M. Mass M. Mass
(A) (B) (C) (D) Same as M. Mass
2 6 3

7. What quantity of ammonium sulphate is necessary for the production of NH3 gas sufficient to
neutralize a solution containing 292 g of HCl ?
[HCl=36.5; (NH4 )2 SO4 =132; NH3 =17]
(A) 272 g (B) 403 g (C) 528 g (D) 1056 g

8. Mohr's salt is dissolved in dil. H2 SO 4 instead of distilled water to


(A) Enhance the rate of dissolution (B) Prevent cationic hydrolysis
(C) Increase the rate of ionisation (D) Increase its reducing strength
9. 0.16 g of dibasic acid required 25 ml of decinormal NaOH solution for complete neutralisation.
The molecular weight of the acid will be
(A) 32 (B) 64 (C) 128 (D) 256
10. To neutralise 20 ml of M / 10 sodium hydroxide, the volume of M / 20 hydrochloric acid required
is
(A) 10 ml (B) 15 ml (C) 20 ml (D) 40 ml

11. KMnO 4 reacts with oxalic acid according to the equation,


2MnO4  5C 2O 42  16H  2Mn2   10CO2  8H2O , here 20 ml of 0.1 M KMnO 4 is equivalent to
(A) 20 ml of 0.5 M H2 C2 O 4 (B) 50 ml of 0.1 M H2 C2 O 4
(C) 50 ml of 0.5 M H2 C2 O 4 (D) 20 ml of 0.1 M H2 C2 O 4

Don’t ever get comfortable when you have the ability to achieve more.
74 REDOX REACTION

12. In order to prepare one litre normal solution of KMnO 4 , how many grams of KMnO 4 are required
if the solution is used in acidic medium for oxidation
(A) 158 g (B) 31.6 g (C) 790 g (D) 62 g

13. A solution containing Na2 CO3 and NaOH requires 300 ml of 0.1 N HCl using phenolpthalein as
an indicator. Methyl orange is then added to the above titrated solution when a further 25 ml of
0.2 N HCl is required. The amount of NaOH present in solution is (NaOH  40,Na2CO3  106)
(A) 0.6 g (B) 1.0 g (C) 1.5 g (D) 2.0 g

14. Which of the following cannot give iodometric titrations


(A) Fe3  (B) Cu2 (C) Pb2  (D) Ag

15. Ca(OH)2  H3PO4  CaHPO4  2H2O the equivalent weight of H3PO4 in the above reaction is
(A) 21 (B) 27 (C) 38 (D) 49

16. 100 ml of 0.1 N hypo decolourised iodine by the addition of x g of crystalline copper sulphate to
excess of KI. The value of ‘x’ is (molecular wt. of CuSO 4 .5H2 O is 250)
(A) 5.0 g (B) 1.25 g (C) 2.5 g (D) 4g

M
17. A solution of 10 ml FeSO 4 was titrated with KMnO 4 solution in acidic medium. The amount
10
of KMnO 4 used will be
(A) 5 ml of 0.1 M (B) 10 ml of 1.1 M
(C) 10 ml of 0.5 M (D) 10 ml of 0.02 M

18. A compound contains atoms of three elements in A, B and C. If the oxidation number of A is +2,
B is +5 and that of C is – 2, the possible formula of the compound is
(A) A 3 (BC4 )2 (B) A 3 (B 4 C)2 (C) ABC2 (D) A 2 (BC3 )2

19. A solution of KMnO4 is reduced to MnO2. The normality of solution is 0.6 N. The molarity is
(A) 1.8 M (B) 0.6 M (C) 0.2 M (D) 0.1 M

20. Number of moles of electrons taken up when 1 mole of NO3– ions is reduced to 1 mole of
NH2OH is :
(A) 2 (B) 4 (C) 5 (D) 6

21. What volume of chlorine at STP is required to liberate all the iodine from 200 ml of 0.2 M KI
solution :
(A) 896 ml (B) 448 ml (C) 224 ml (D) 672 ml

22. When one gram mole of KMnO4 reacts with HCl, the volume of chlorine liberated at NTP will
be :
(A) 11.2 litres (B) 22.4 litres
(C) 44.8 litres (D) 56.0 litres

CatalyseR Eduventures (India) Pvt. Ltd.


REDOX REACTION 75

23. Hydrogen peroxide in aqueous solution decomposes on warming to give oxygen according to the

equation 2H2O2(aq.)   2H2O(  ) + O2(g) under conditions where one mole of gas
occupies 24 dm3, 100 cm3 of XM solution of H2O2 produces 3dm3 of O2. X is thus :
(A) 2.5 (B) 1 (C) 0.5 (D) 0.25

24. In an experiment 50 ml of 0.1 M solution of a salt reacted with 25 ml of 0.1 M solution of sodium
sulphite. The half equation for the oxidation of sulphite ion is –

SO32– (aq.) + H2O(l)   SO42– (aq.) + 2H+ (aq.) + 2e–


If the oxidation number of metal in the salt was 3, what would be the new oxidation number of
metal ?
(A) 0 (B) 1 (C) 2 (D) 4

25. When BrO3– ion reacts with Br– ion in acid solution Br2 is liberated the equivalent weight of
KBrO3 in this reaction is :
(A) M/8 (B) M/3 (C) M/5 (D) M/6
[where M is the molar mass ]
26. A solution contains both Na2CO3 and NaHCO3 was treated with excess of CaCl2 solution and
filtered. The precipitate weighed m1 grams. On adding NaOH in drops to the filtrate avoiding
excess, a further m2 grams was precipitated. If after adding excess CaCl2, the solution (had not
been filtered) but was simply boiled and then filtered, what would be the total weight of the
precipitate ?
(A) (m1 + m2) grams (B) (m1 + m2/2) grams
(C) (m1 + m2) /2 grams (D) (m2+m1/2) grams

27. 2 grams of a gas mixture of CO and CO2 on reaction with excess I2O5 yield 2.54 grams of I2.
What would be the weight % of CO in the original mixture ?
(A) 70.1 (B) 75.3
(C) 68.4 (D) 80.7
28. One gram of a mixture of Na2CO3 and NaHCO3 consumes y gram equivalents of HCl for
complete neutralisation. One gram of the mixture is strongly heated, then cooled and the residue
treated with HCl. How many gram equivalents of HCl would be required for complete
neutralisation :
(A) 2y gram equivalent (B) y gram equivalents
(C) 3y/4 gram equivalents (D) 3y/2 gram equivalents

29. An iodide solution, ½ litre containing 0.664 grams of KI per litre, required 100 ml of a certain
solution of Ce4+ ion for complete reaction. What is the normality of the Ce4+ solution ?
(A) 0.04 (B) 0.004
(C) 0.02 (D) none of these

Don’t ever get comfortable when you have the ability to achieve more.
76 REDOX REACTION

30. A solution contains Na2CO3 and NaHCO3. 10 ml of the solution required 2.5 ml of 0.1 M H2SO4
for neutralisation using phenolphthalein as indicator. Methyl orange is then added when a further
2.5 ml of 0.2 M H2SO4 was required. The amount of Na2CO3 and NaHCO3 in 1 litre of the
solution is :
(A) 5.3 g & 4.2 g (B) 3.3 g & 6.2 g (C) 4.2 g & 5.3 g (D) 6.2 g & 3.3 g

31. 0.7 g of a sample of Na2CO3. x H2O were dissolved in water and the volume was made to 100
ml . 20 ml of this solution required 19.8 ml of N/10 HCl for complete neutralization. The value of
x is :
(A) 7 (B) 3 (C) 2 (D) 5

32. What will happen if the solution of potassium chromate reacts with excess amount of nitric acid
(A) Cr reduces in the oxidation state +3 from CrO42–.

(B) Cr oxidises in the oxidation state +7 from CrO42–.

(C) Cr+3 and Cr2O72– will be formed.

(D) Cr2O72– and H2O will be formed.

33. CuSO4 reacts with excess amount of KI, followed by solution of Na2S2O3. In this process which
of following statement is incorrect :
(A) CuI will be formed (B) Evolved I2 will be reduced
(C) Na2S2O3 will be oxidised (D) CuI2 will be formed

34. The oxidation state of chromium in the final product formed by the reaction between KI and
acidified potassium dichromate solution is :
(A) + 4 (B) + 6 (C) + 2 (D) + 3

35. One mole of N2H4 loses ten moles of electrons to form a new compound Y. Assuming that all the
nitrogen appears in the new compound, what is the oxidation state of nitrogen in Y ?
[There is no change in the oxidation state of hydrogen]
(A) -1 (B) -3 (C) +3 (D) +5



CatalyseR Eduventures (India) Pvt. Ltd.


REDOX REACTION 77

EXERCISE # 02 ADVANCED LEVEL EXERCISE


(COMPREHENSION TYPE, MULTIPLE TYPE, MATRIX MATCH TYPE, INTEGER TYPE)

MULTIPLE ANSWERS CORRECT TYPE

1. Which of the following represent redox reactions :


(A) Cr2 O7 2   2OH 
 2CrO 4 2   H2 O
(B) 2Cr2 O4 2   2H 
 Cr2 O7 2   H2 O
(C) 2MnO4   3Mn2   4OH  5MnO2  2H2 O
(D)  Cu  Cu2 
2Cu 

2. When  NH4  2 Cr2O7 is heated :


(A) there is oxidation of N (B) there is reduction of Cr
(C) net reaction is disproportionations (D) net reaction is neutralisation

3. Which of the following are disproportionation reaction?


Al OEt  
(A) 2RCHO 
3
RCOOCH2R (B) 4H3PO3   3H3PO4  PH3
 
(C) NH4NO3  N 2O  2H2O (D) PCl5   PCl3  Cl2

4. For the reaction : H3PO4  Ca  OH2 


 CaHPO4  2H2O
1 mol 1 mol
Which are true statements :
(A) equivalent weight of H3PO4 is 49
(B) resulting mixture is neutralised by 1 mol of KOH
(C) CaHPO4 is an acid salt

(D) 1 mol of H3PO4 is completely neutralised by 1.5 mol of Ca  OH2 .

5.  PH3  2H3PO3 . In this reaction :


3H3PO2 
(A) H3PO2 undergoes disproportionation (B) equivalent weight of H3PO2 is 22
(C) equivalent weight of H3PO2 is 49.5
(D) NaH2PO2 is not acid salt.

6. H2 C2 O 4 and NaHC 2O 4 behave as acids as well as reducing agents. which are correct
statement?
(A) equivalent weight of H2 C2 O 4 and NaHC2O 4 are equal to their molecular weights when
behaving as reducing agents
(B) 100 ml of 1 N solution of each is neutralised by equal volume of 1M Ca  OH2

(C) 100 ml of 1 N solution of each is neutralised by equal volume of 1N Ca  OH2

(D) 100 ml of 1 M solution of each is oxidised by equal volumes of 1M KMnO 4

Don’t ever get comfortable when you have the ability to achieve more.
78 REDOX REACTION

7. Which of the following are primary standard substances ?


(A) Na2 CO3 .10H2 O (B) NaOH (C) Na2B 4 O7 .10H2O (D) KMnO 4

8. Which of the following statements are correct?


(A) the point at which an equivalent amount of the titrant is added is called the equivalence
point.
(B) the point at which the reaction is observed to be complete is called the end point
(C) at the end point of a reaction there is no change in the properties of the solution
(D) at the equivalence point of a reaction the stoichiometric amount of the titrant is not added

9. Which of following will be present in the solution formed when 50 mL of 0.1 M HCl is mixed with
50 mL of 0.1 M NaOH ?
(A) 4.5 m mol of H (B) 0.05 m mol of OH-
(C) 0.05 M NaCl (D) 10 7 M of H ion

10. During the titration of a mixture of Na2 CO3 and NaHCO3 against HCl,
(A) phenolphthalein is used to detect the first end point
(B) phenolphthalein is used to detect the second end point
(C) methyl orange is used to detect the second end point
(D) methyl red is used to detect the first end point

11. 1 mol of H2 SO 4 will exactly neutralize

(A) 2 mol of ammonia (B) 1 mol of Ba  OH2

(C) 0.5 mol of Ba  OH2 (D) 2 mol of KOH

12. A solution of Na 2S 2 O3 is standardized iodometrically against 0.1262 g of KBrO3 . This process


requires 0.45 mL of Na 2S 2 O3 solution. What is the strength of the Na 2S 2 O3 ?
(A) 0.2 M (B) 0.1 M (C) 0.05 N (D) 0.1 N

13. Which of the following are redox reactions?


(A) Mg  N2  Mg3N2

(B) K 4 Fe  CN6   H2SO4  H2O 


 K2SO4  CO  FeSO4  NH4 2 SO4

(C) I 2  3Cl2 
 ICl3

(D)  Cu  NH3  4  SO4


CuSO4  NH3 

14. Which of the following have been arranged in order of decreasing oxidation number of sulphur?
(A) H2 S 2 O7  Na2 S 4 O 6  Na2 S 2 O3  S 8 (B) SO2  SO4 2   SO3 2   HSO 4 
(C) H2SO5  H2 SO3  SCl2  H2 S (D) H2 SO4  SO2  H2 S  H2 S 2O8

15. 0.6 mole of K2Cr2O7 can oxidize –


(A) 3.6 moles of FeSO4 to Fe2(SO4)3 (B) 0.1 moles of FeSO4 to Fe2(SO4)3
(C) 0.05 moles of Sn2+ to Sn4+ (D) 1.8 mole of Sn2+ to Sn4+

CatalyseR Eduventures (India) Pvt. Ltd.


REDOX REACTION 79

16. In which of the following reaction (s) H2SO4 act as an acid as well as an oxidant?
H2SO4(Conc.)
(A) C12H22O11 12 C
–11H2O
(B) Zn + H2SO4 ZnSO4 + H2
(C) Cu + 2H2SO4  CuSO4 + SO2 + 2H2O
(D) NaHCO3 + H2SO4  NaHSO4 + H2O + CO2

17. Identify the correct statement (s) regarding the oxidation of N2H5 by Cr2O72– in acid medium.
(A) 10 ml 0.2 M dichromate is required for the complete oxidation of 12 ml 0.25 M N2H5
(B) 10 ml 0.2 M dichromate is required for the complete oxidation of 10 ml 0.6 N N2H5
(C) Number of moles of N2 liberated per mole of dichromate is 1.5
(D) One mole of N2H5 is 3 equivalents

18. If 1 mole of H3PO4 is reacted with 1 mole of X (OH)2 as –


H3PO4 + X(OH)2  X HPO4 + 2H2O then
mol wt.
(A) The equivalent weight of base is
2
98
(B) The eq. wt. of H3PO4 is
3
(C) The resulting solution is required 1 mole NaOH for complete neutralisation
(D) 1 mole of X(OH)2 more required for complete neutralisation of X HPO4

19. Which of the following is/are correct ?


(A) The number of peroxo linkages in H2SO5 is one
(B) The number of peroxo linkages in H2S2O8 is one
(C) The number of peroxo linkages in CrO5 is two
(D) The number of peroxo linkages in FeO24 is two

20. In which of the following species the O.S. of the carbon is/are equal to + 2 only
(A) H–C  N (B) C
H – N (C) O
C (D) O=C=C=C=O

21. Lowest possible oxidation states of Mn, Cr, Cl and Os are +2, +3, –1 and zero respectively.
Which of the given species require least number of moles to oxidise 25 ml of 0.5 (M) Fe2+
solution?
(A) OsO4 (B) MnO4 (C) ClO4 (D) Cr2O 72 

22. For complete oxidation of FeC2O4 it requires.


1
(A) 3/5 mole of acidified KMnO4 solution (B) mole of acidified K2Cr2O7 solution
2
(C) 2 mole of HNO3 (D) 2 mole of Sn2+ solution.

Don’t ever get comfortable when you have the ability to achieve more.
80 REDOX REACTION

MATCH THE COLUMN TYPE


23. Match the column -

COLUMN – I COLUMN – II
Reactant n-factor
(a) H3PO3 (i) 11/3
(b) H3PO4 (ii) 2
(c) (iii) 3
3MnO4–  2Mn2+ + Mn+6

(d)
FeC2O4  Fe3+ + 2CO2

(e)
2H2O2 
 O2 + 2H2O
(A) (a) - (ii) , (b) - (iii) , (c) - (i) , (d) - (iii), (e)-(ii)
(B) (a) - (i) , (b) - (ii) , (c) - (iii) , (d) - (ii), (e)-(i)
(C) (a) - (ii) , (b) - (i) , (c) - (ii) , (d) - (iii), (e)-(ii)
(D) (a) - (iii) , (b) - (ii) , (c) - (i) , (d) - (ii), (e)-(iii)

24. Match the column –

COLUMN – I COLUMN – II
(a) HClO 4 (i) Oxidising agent
(b) H2S (ii) Reducing agent
(c) Na 2 S 2 O 7 (iii) Oxidising as well as reducing agent
(d) SO 2
(e) HNO 2

(A) (a) - (ii) , (b) - (iii) , (c) - (i) , (d) - (iii), (e)-(ii)
(B) (a) - (i) , (b) - (ii) , (c) - (iii) , (d) - (ii), (e)-(i)
(C) (a) - (ii) , (b) - (i) , (c) - (ii) , (d) - (iii), (e)-(ii)
(D) (a) - (i) , (b) - (ii) , (c) - (ii) , (d) - (iii), (e)-(iii)

25. Match the column –


COLUMN – I COLUMN – II

(A) SCN  SO –2
4 + HCN (P) Disproportionation reaction
(B) KClO3  KClO4 + KCl (Q) n-factor = 6
(C) K 2Cr2O7  H 
 Cr2  SO4 3  Cr2O3 (R) molecular mass
Equivalent mass =
11
(D) FeS2  Fe3+ + SO2 (S) Two elements show change in
oxidation state

CatalyseR Eduventures (India) Pvt. Ltd.


REDOX REACTION 81

26. Match the column –

COLUMN – I COLUMN – II
(A) 2 mole of electrons transfer (P) C 2 O 42  CO2
(B) (Q) 2+
3 mole of electrons transfer Fe3O4  Fe
(C) 4 mole of electrons transfer (R) 3+
CrO42  Cr
(D) Reduction reactions (S) MnO4  MnF4 

27. Match the half-reactions (in column- I) with change in oxidation number (in column- II) :

COLUMN – I COLUMN – II
(A) 
Cl  ClO 
4
(P) 2
(B) Cr 3  
 CrO5 (Q) 8
(C) H2 O2  O2 (R) 0
(D) CrO 22  CrO24 (S) 3

COMPREHENSION TYPE
COMPREHENSION # 01 (Ques. 28 to 30)

A sequential oxidation reduction represented as follows

5+ (I) m+ (II) n+ (III) p+


A A A A

Sn2+ N2H4 I2

Sn4+ N2 I–
–2 5+ –3 2+
As per reaction (I), for 10 moles A , 5 × 10 moles of Sn was required.
For Am+ so produced, as per reaction (II) 2.5 × 10–3 moles of N2H4 was required.
For An+ so produced, as per reaction (III) 5 × 10–3 moles of I2 was required 

28. The value of m is -


(A) + 4 (B) +3 (C) +2 (D) +1

29. The value of n is -


(A) + 4 (B) +3 (C) +2 (D) +1

30. The value of p is -


(A) + 4 (B) +3 (C) +2 (D) +1

Don’t ever get comfortable when you have the ability to achieve more.
82 REDOX REACTION

COMPREHENSION # 02 (Ques. 31 to 34)

Redox reactions one those in which oxidation and reduction take place simultaneously.
Oxidant can gain electron where as reductant can lose electron easily. The oxidation
state of any element can never be in fraction. If oxidation number of any element comes
out be in fraction, it is average oxidation number of that element which is present in
different oxidation states.

31. The oxidation no. of Fe in Fe3O4 is -


(A) 2 and 3 (B) 8/3 (C) 2 (D) 3

1
N 3 1 2 3
32. N–H in this compound HN3 (Hydrazoic acid) oxidation no. of N , N and N are -
N
2
(A) 0, 0, 3 (B) 0, 0, –1 (C) 1, 1, –3 (D) – 3, –3, –3.

33. Eq. wt. of chlorine molecule in the equation


3Cl2 + 6NaOH  5NaCl + NaClO3 + 3H2O
(A) 42.6 (B) 35.5 (C) 59.1 (D) 71

34. The equivalent weight of Na2S2O3 in the following reaction is Na2S2O3 + 2HCl  2NaCl + H2O +
S + SO2 Given : molar mass of Na2S2O3 = M
M M 3M
(A) M (B) (C) (D)
2 3 4

COMPREHENSION # 03 (Ques. 35 to 39)

In the higher temperature bleaching powder disproportionate to some extent and yield Ca(ClO3)2
and CaCl2. The 10 gm of a pyrollusite sample of 26.1% purity is warmed with 240 ml of 0.5 (M)
HCl, the Cl2 (g) so produced is passed through 3.74 gm of slaked lime (hydrated CaO) containing
40% moisture at 40ºC to produce bleaching powder. The bleaching powder so produced is the
"moist bleaching powder". To produce "dry bleaching powder" the "moist bleaching powder" is
heated at 80ºC where Ca(OCl)Cl disproportionate to some extent. "Dry bleaching powder" so
produced is divided into two equal parts. To the first part excess of HCl and excess of KI solution
added I2 so produced require 300 ml of 10–3 (M) Na2S2O3 solution. To the second part excess of
CH3COOH and excess of KI solution is added the iodine so produced require 200 ml of 10–3 (M)
Na2S2O3 solution. It is also seen during drying of moist bleaching powder beside
disproportionation of Ca(OCl)Cl other side reactions also occur "without any gaseous products."

35. The mass of moist bleaching powder sample produced is


(A) 5.87 gm (B) 5.306 gm
(C) 3.81 gm (D) 4.67 gm

CatalyseR Eduventures (India) Pvt. Ltd.


REDOX REACTION 83

36. What is the amount of "Dry bleaching powder" produced?


(A) 5.87 gm (B) 5.306 gm
(C) 4.374 gm (D) 4.67 gm

37. What is the percentage of available chlorine in the "dry bleaching powder"?
(A) 55.9% (B) 48.7%
(C) 75.7 % (D) 0.32%

38. What is the reason for the differences in the volume of Na2S2O3 solution for the two equal part of
the dry bleaching powder sample?
(A) There is no definite reason it was an experimental error
(B) In the first part both I2 and Cl2 consume Na2S2O3 solution whereas for second part only I2
consume Na2S2O3 solution.
(C) For the Ist part in the presence of excess HCl, Ca(ClO3)2 also liberate I2 which consume
excess of Na2S2O3 solution whereas for the second part only Ca(OCl) Cl liberate I2.
(D) None of these

39. What is the % of Ca(ClO3)2 impurity in the "dry bleaching powder" sample?
(A) 0.079% (B) 0.039%
(C) 1% (D) 0.001%
INTEGER TYPE

40. The number of peroxolinkages present in CrO5 is …….

41. The reaction Cl2 (g) + S 2 O32  SO42  + Cl– is to be carried out in basic medium. Starting
with 0.15 mole of Cl2, 0.010 moles of S 2 O32 and 0.30 moles of OH– , how many milli moles of
OH– will be left in solution after the reaction is complete ?
Assume no other reaction occurs.

42. How many oxyanions among the following show disproportionation?


ClO  ,ClO2 ,ClO3  ,ClO4 

43. In the following reaction, hydrazine is oxidized to N2


N2H4  OH  N2  H2O  e 
The equivalent mass of N2H4 (hydrozine) is :

44. Nitrobenzene  C6H5NO2  can be reduced to aniline  C6H5NH2  by electrolytic reduction; the

 molecular mass 
equivalent mass of nitrobenzene will be equal to   . The value of n is:
 n 

45. 6  10 3 mole K 2 Cr2O7 reacts completely with 9  10 3 mole xn  to give XO3 and Cr 3  . The
value of n is:

Don’t ever get comfortable when you have the ability to achieve more.
84 REDOX REACTION

46. If four mole of Br2 undergo a loss and gain of six mole electrons to form two new oxidation state
of Br2 . How much Br2 mole acts as reductant?

47. The stoichiometric coefficient of blue perchromate in its reaction with H2 SO 4 is ………

48. The ratio of oxygen atom having 2 and  1 oxidation numbers in S 2 O82  is ………...

49. 1 mole of Cu2 S reduces how many moles of KMnO 4 ? If the redox reaction is:
Cu2S  KMnO 4  H2 SO 4 
 CuSO4  MnSO 4  K 2 SO 4  H2 O

50. The difference in the oxidation number of the two types of sulphur atoms in Na2 S 4 O6 is:



CatalyseR Eduventures (India) Pvt. Ltd.


REDOX REACTION 85

EXERCISE # 03 SUBJECTIVE EXERCISE # 01

1. How many ml of 0.1 N HCl are required to react completely with 1 g mixture of Na2CO3 and
NaHCO3 containing equimolar amounts of two?

2. 0.5 g of fuming H2SO4 (oleum) is diluted with water. The solution requires 26.7 ml of 0.4 N NaOH
for complete neutralization. Find the % of free SO3 in the sample of oleum.

3. 10 g CaCO3 were dissolved in 250 ml of M HCl and the solution was boiled. What volume of 2 M
KOH would be required to equivalence point after boiling? Assume no change in volume during
boiling.

4. 1.64 g of a mixture of CaCO3 and MgCO3 was dissolved in 50 mL of 0.8 M HCl. The excess of
acid required 16 mL of 0.25 M NaOH for neutralization. Calculate the percentage of CaCO3 and
MgCO3 in the sample.

5. 1.5 g of chalk were treated with 10 ml of 4N – HCl. The chalk was dissolved and the solution
made to 100 ml 25 ml of this solution required 18.75 ml of 0.2 N – NaOH solution for complete
neutralisation. Calculate the percentage of pure CaCO3 in the sample of chalk?

6. A solution contains Na2CO3 and NaHCO3. 20ml of this solution required 4ml of 1N HCl for
titration with Ph indicator. The titration was repeated with the same volume of the solution but with
MeOH. 10.5 ml of 1N HCl was required this time. Calculate the amount of Na2CO3 & NaHCO3.

7. A solution contains a mix of Na2CO3 and NaOH. Using Ph as indicator 25ml of mix required 19.5
ml of 0.995 N HCl for the end point. With MeOH, 25 ml of the solution required 25ml of the same
HCl for the end point. Calculate gms/L of each substance in the mix.

N
8. 200 ml of a solution of mixture of NaOH and Na2CO3 was first titrated with Ph and HCl. 17.5
10
ml of HCl was required for end point. After this MeOH was added and 2.5 ml of same HCl was
again required for next end point. Find out amounts of NaOH and Na2CO3 in the mix.

9. A solution contains Na2CO3 and NaHCO3. 10ml of this requires 2ml of 0.1M H2SO4 for
neutralisation using Ph indicator. MeOH is then added when a further 2.5 ml of 0.2 M H2SO4 was
needed. Calculate strength of Na2CO3 and NaHCO3.

10. A sample containing Na2CO3 & NaOH is dissolved in 100ml solution. 10ml of this solution
requires 25ml of 0.1N HCl when Ph is used as indicator. If MeOH is used as indicator 10ml of
same solution requires 30ml of same HCl. Calculate % of Na2CO3 and NaOH in the sample.

Don’t ever get comfortable when you have the ability to achieve more.
86 REDOX REACTION

11. It requires 40.05 ml of 1M Ce4+ to titrate 20ml of 1M Sn2+ to Sn4+. What is the oxidation state of
the cerium in the product.

12. A volume of 12.53 ml of 0.05093 M SeO2 reacted with exactly 25.52 ml of 0.1M CrSO4. In the

reaction, Cr2+ was oxidized to Cr3+. To what oxidation state was selenium converted by the
reaction.

13. Potassium acid oxalate K2C2O4·3HC2O4·4H2O can be oxidized by MnO4– in acid medium.
Calculate the volume of 0.1M KMnO4 reacting in acid solution with one gram of the acid oxalate.

14. A 1.0g sample of H2O2 solution containing x % H2O2 by mass requires x cm3 of a KMnO4
solution for complete oxidation under acidic conditions. Calculate the normality of KMnO4
solution.

15. Metallic tin in the presence of HCI is oxidized by K2Cr2O7 to stannic chloride, SnCl4. What
volume of deci-normal dichromate solution would be reduced by 1g of tin.

16. 5g sample of brass was dissolved in one litre dil. H2SO4. 20 ml of this solution were mixed with

KI, liberating I2 and Cu+ and the I2 required 20 ml of 0.0327 N hypo solution for complete
titration. Calculate the percentage of Cu in the alloy.

17. A 1.0 g sample of Fe2O3 solid of 55.2% purity is dissolved in acid and reduced by heating the
solution with zinc dust. The resultant solution is cooled and made upto 100.0 mL. An aliquot of
25.0 mL of this solution requires 17.0 mL of 0.0167 M solution of an oxidant for titration. Calculate
the number of moles of electrons taken up by the oxidant in the reaction of the above titration.

18. 0.84 g iron ore containing x percent of iron was taken in a solution containing all the iron in
ferrous condition. The solution required x ml of a dichromatic solution for oxidizing the iron
content to ferric state. Calculate the strength of dichromatic solution.

19. The neutralization of a solution of 1.2 g of a substance containing a mixture of H2C2O4. 2H2O,
KHC2O4. H2O and different impurities of a neutral salt consumed 18.9 ml of 0.5 N NaOH
solution. On titration with KMnO4 solution, 0.4 g of the same substance needed 21.55 ml of 0.25
N KMnO4. Calculate the % composition of the substance.

20. 50gm of a sample of Ca(OH)2 is dissolved in 50ml of 0.5N HCl solution. The excess of HCl was
titrated with 0.3N – NaOH. The volume of NaOH used was 20cc. Calculate % purity of Ca(OH)2.

CatalyseR Eduventures (India) Pvt. Ltd.


REDOX REACTION 87

21. One gm of impure sodium carbonate is dissolved in water and the solution is made up to 250ml.
To 50ml of this made up solution, 50ml of 0.1N HCl is added and the mix after shaking well
required 10ml of 0.16N NaOH solution for complete titration. Calculate the % purity of the sample.

22. A sample of Mg was burnt in air to give a mix of MgO and Mg3N2. The ash was dissolved in 60
meq HCl and the resulting solution was back titrated with NaOH. 12 meq of NaOH were required
to reach end point. An excess of NaOH was then added and the solution distilled. The NH3
released was then trapped in 10 meq of second acid solution. Back titration of this solution
required 6 meq of the base. Calculate the % of Mg burnt to the nitride.

23. What amount of substance containing 60% NaCl, 37% KCl should be weighed out for analysis so
that after the action of 25 ml of 0.1N AgNO3 solution, excess of Ag+ is back titrated with 5 ml of
NH4SCN solution? Given that 1 ml of NH4SCN = 1.1 ml of AgNO3.

24. 5g of pyrolusite (impure MnO2) were heated with conc. HCl and Cl2 evolved was passed through
N
excess of KI solution. The iodine liberated required 40 mL of hypo solution. Find the % of
10
MnO2 in the pyrolusite.

25. A sample of calcium carbonate contains impurities which do not react with a mineral acid. When 2
grams of the sample were reacted with the mineral acid, 375 ml of carbon dioxide were obtained
at 27°C and 760 mm pressure. Calculate the % purity of the sample of CaCO3?



Don’t ever get comfortable when you have the ability to achieve more.
88 REDOX REACTION

EXERCISE # 04 SUBJECTIVE EXERCISE # 02

1. One gram of an alloy of aluminium and magnesium when heated with excess of dil. HCl forms
magnesium chloride, aluminium chloride and hydrogen. The evolved hydrogen collected over
mercury at 0°C has a volume of 1.2 litres at 0.92 atm pressure. Calculate the composition of the
alloy.

2. 10 gm of a mixture of anhydrous nitrates of two metal A & B were heated to a constant weight &
gave 5.531 gm of a mixture of the corresponding oxides. The equivalent weights of A & B are
103.6 & 31.8 respectively. What was the percentage of A in the mixture.

3. A mixture of FeO and Fe2O3 is reacted with acidified KMnO4 solution having a concentration of
0.2278 M, 100 ml of which was used. The solution was then titrated with Zn dust which converted
Fe3+ of the solution to Fe2+. The Fe2+ required 1000 ml of 0.13 M K2Cr2O7 solution. Find the %
of FeO & Fe2O3.

4. 50ml of a solution, containing 0.01 mole each Na2CO3, NaHCO3 and NaOH was titrated with N-
HCl. What will be the titre readings if
(a) only Ph is used as indicator.
(b) only MeOH is used as indicator from the beginning.
(c) MeOH is added after the first end point with Ph.

5. A 0.517g sample containing Ba(SCN)2 was dissolved in a bicarbonate solution. 50.0 mL of 0.107
N iodine was added, and the mixture was allowed to stand for five minutes. The solution was then
acidified, and the excess I2 was titrated with 16.3 mL of 0.0965 M sodium thiosulphate. Write a

balanced equation for the oxidation of SCN into SO24 and HCN. Calculate the percent
Ba(SCN)2 in the sample.

6. An acid solution of a KReO4 sample containing 26.83 mg of combined rhenium was reduced by
passage through a column of granulated zinc. The effluent solution, including the washings from
the column, was then titrated with 0.10 N KMnO4. 11.45 mL of the standard permanganate was

required for the re-oxidation of all the rhenium to the perrhenate ion, ReO4–. Assuming that
rhenium was only element reduced. What is the oxidation state to which rhenium was reduced by
the Zn column. (Atomic mass of Re = 186.2)

7. Chrome alum K2SO4.Cr2(SO4)3.24 H2O is prepared by passing SO2 gas through an aqueous
solution of K2Cr2O7 acidified with dilute sulphuric acid till the reduction is complete. The alum is
crystallized followed by filtration/centrifugation. If only 90% of the alum can be recovered from the
above process, how much alum can be prepared from 10kg of K2Cr2O7? Give the number of
moles of electrons supplied by SO2 for reducing one mole of K2Cr2O7.

CatalyseR Eduventures (India) Pvt. Ltd.


REDOX REACTION 89

8. A substance of crude copper is boiled in H2SO4 till all the copper has reacted. The impurities are
inert to the acid. The SO2 liberated in the reaction is passed into 100 mL of 0.4 M acidified
KMnO4. The solution of KMnO4 after passage of SO2 is allowed to react with oxalic acid and
requires 23.6 mL of 1.2 M oxalic acid. If the purity of copper is 91%, what was the weight of the
sample.

9. A 1.87gm. sample of chromite ore(FeO.Cr2O3) was completely oxidized by the fusion of


peroxide. The fused mass was treated with water and boiled to destroy the excess of peroxide.
After acidification the sample was treated with 50ml. of 0.16M Fe2+. In back titration 2.97 ml of
0.005 M barium dichromate was required to oxidize the excess iron (II). What is the percentage of
chromite in the sample?

10. 25 mL of a solution containing HCl was treated with excess of M/5 KIO3 and KI solution of
unknown concentration where I2 liberated is titrated against a standard solution of 0.021M
Na2S2O3 solution whose 24 mL were used up. Find the strength of HCl and volume of KIO3
solution consumed.

11. A 10g sample of only CuS and Cu2S was treated with 100 mL of 1.25 M K2Cr2O7. The products

obtained were Cr3+, Cu2+ and SO2. The excess oxidant was reacted with 50 mL of Fe2+

solution. 25 ml of the same Fe2+ solution required 0.875M acidic KMnO4 the volume of which
used was 20 mL. Find the % of CuS and Cu2S in the sample.

12. H2O2 is reduced rapidly by Sn2+, the products being Sn4+ & water. H2O2 decomposes slowly at

room temperature to yield O2 & water. Calculate the volume of O2 produced at 20 C & 1.00 atm

when 200g of 10.0 % by mass H2O2 in water is treated with 100.0 ml of 2.00 M Sn2+ & then the
mixture is allowed to stand until no further reaction occurs.

13. 0.6213 g of sample contains an unknown amount of As2O3. The sample was treated with HCl
resulting in formation of AsCl3(g) which was distilled into a beaker of water. The hydrolysis

reaction is as follows AsCl3 + 2H2O  HAsO2 + 3H+ + 3Cl


The amount of HAsO2 was determined by titration with 0.04134 M I2, requiring 23.04 mL to reach

the equivalence point. The redox products in the titration were H3AsO4 and I . Find the amount
of KMnO4 needed to oxidize As in As2O3 to its maximum possible oxidation state in acidic
medium.

Don’t ever get comfortable when you have the ability to achieve more.
90 REDOX REACTION

14. A sample of steel weighing 0.6 gm and containing S as an impurity was burnt in a stream of O2,

when S was converted to its oxide SO2. SO2 was then oxidized to SO4– – by using H2O2
solution containing 30ml of 0.04 M NaOH. 22.48 ml of 0.024 M HCl was required to neutralize the
base remaining after oxidation. Calculate the % of S in the sample.

15. A 10gm mixture of Cu2S and CuS was treated with 200 ml of 0.75 M MnO4– in acid solution,

producing SO2, Cu2+ & Mn2+. The SO2 was boiled off and the excess MnO4– was titrated with

175 ml of 1M Fe2+ solution. Calculate the % CuS in the original mixture.

16. 3.3 gm of a sample of Anhydrous CuSO4 was dissolved in water and made to 250ml. 25 ml of
this solution after taking usual precautions was treated with a little excess of KI solution. A white
ppt. of Cu2I2 and iodine was evolved. The iodine so evolved required 24.6 ml of hypo solution
containing 20gm of (Na2S2O3 · 5H2O) per litre. What is the purity of CuSO4 solution.

17. A certain sample of coal contained some iron pyrite (FeS2) – a pollution causing impurity. When
the coal was burned iron(II) was oxidised and SO2 was formed. The SO2 was reacted with

NaOH when sodium sulphite and water was formed. On a particular day 103 kg of coal was
burned and it required 4 litres of 5M NaOH for the treatment of SO2. What was the percentage of
pyrite in the coal. What was the percentage of sulphur in the coal.

18. In the presence of fluoride ion, Mn2+ can be titrated with MnO4—, both reactants being
converted to a complex of Mn(III). A 0.545 g sample containing Mn3O4 was dissolved and all

manganese was converted to Mn2+. Titration in the presence of fluoride ion consumed 31.1 ml of
KMnO4 that was 0.117 N against oxalate.

(a) write a balanced chemical equation for the reaction, assuming that the complex is MnF4—.
(b) what was the % of Mn3O4 in the sample?

19. Calculate the % of MnO2 in a sample of pyrolusite ore, 1.5 g which was made to react with 10 g.

of Mohr’s salt (FeSO4.(NH4)2SO4. 6H2O) and dilute H2SO4. MnO2 was converted Mn2+. After
the reaction the solution was diluted to 250 ml and 50 ml of this solution, when titrated with 0.1 N
K2Cr2O7, required 10 ml of the dichromate solution.

20. 30cc of a solution containing 9.15gm of a salt KXHY (C2O4)Z · nH2O per litre required 27cc of
0.12 N NaOH for neutralization. The same quantity of solution was also found to require 36cc of
0.12 N KMnO4 solution for complete oxidation. Calculate X, Y, Z and n.



CatalyseR Eduventures (India) Pvt. Ltd.


REDOX REACTION 91

EXERCISE # 05 KVPY, OLYMPIADS QUESTIONS

1. The number of electrons required to reduce chromium completely in Cr2O72  to Cr 3  in acidic


medium, is [KVPY 2015]
(A) 2 (B) 3 (C) 4 (D) 5

2. The oxidation number of sulphur is 4 in [KVPY 2016]


(A) H2S (B) CS2 (C) Na 2 SO 4 (D) Na 2 SO3

3. A redox reaction among the following is - [KVPY 2016]


(i) CdCl2  2KOH  Cd OH 2  2KCl

(ii) BaCl2  K 2 SO4  BaSO 4  2KCl


(iii) CaCO3  CaO  CO2
(iv) 2Ca  O 2  2CaO
(A) (i) (B) (ii) (C) (iii) (D) (iv)

4. The pair of compounds that will not react with each other in an aqueous solution, at room
temperature is [NSEC 2014]
(A) FeCl3 ,SnCl2 (B) HgCl2 ,SnCl2 (C) FeCl2 ,SnCl2 (D) FeCl3 ,KI

5. I. 5H2 O2  2MnO4  6H  2Mn2  5O2  8H2 O


II. H2 O2  Ag2 O  2Ag  H2 O  O2
The role of hydrogen peroxide in the above reactions is [NSEC 2014]
(A) oxidizing in I and reducing in II (B) reducing in I and oxidising in II
(C) reducing in I as well as in II (D) oxidising in I as well as in II

6. Absorbance of a chlorophyll solution measured at 660 nm at 25o C using a 1 cm cell was found
to be 0.4. The same solution is heated up to 35o C and absorbance is measured once again
under the same condition. The observed absorbance will be [NSEC 2014]
(A) 0.2 (B) 0.4 (C) 0.6 (D) 0.8

7. The species which is unable to show disproportionation reaction is [NSEC 2014]


(A) ClO 
3 (B) ClO 
4 (C) ClO 
2 (D) ClO 

8. In the redox reaction


2MnO4  5C 2O 42  16H  2Mn2   10CO2  8H2O
20 mL of 0.1 M KMnO 4 react quantitatively with [NSEC 2015]
(A) 20 mL of 0.1 M oxalate (B) 40 mL of 0.1 M oxalate
(C) 50 mL of 0.25 M oxalate (D) 50 mL of 0.1 M oxalate


Don’t ever get comfortable when you have the ability to achieve more.
92 REDOX REACTION

EXERCISE # 06 JEE (MAIN) CORNER

1. Which of the following chemical reactions depicts the oxidizing behaviour of H2 SO 4 ? [2006]
(A) 2HI  H2 SO 4  I2  SO2  2H2 O
(B) Ca  OH 2  H2 SO4  CaSO 4  2H2O

(C) NaCl  H2SO 4  NaHSO 4  HCl


(D) 2PCl5  H2 SO4 
 2POCl3  2HCl  SO2 Cl2

2. What products are expected from the disproportionation reaction of hypochlorous acid? [2006]
(A) HClO3 and Cl2 O (B) HClO2 and HClO 4
(C) HCl and Cl2 O (D) HCl and HClO3

3. Amount of oxalic acid present in a solution can be determined by its titration with KMnO4 solution
in the presence of H2 SO 4 . The titration gives unsatisfactory result when carried out in the
presence of HCl, because HCl : [2008]

(A) Furnishes H ions in addition to those from oxalic acid.
(B) Reduces permanganate to Mn2+.
(C) Oxidises oxalic acid to carbon dioxide and water.
(D) Gets oxidised by oxalic acid to chlorine.

z
4. Consider the following reaction xMnO4   y C2O 4 2  zH  xMn2   2y CO 2  H2 O
2
The values of x, y and z are respectively. [2013]
(A) 2, 5 and 8 (B) 2, 5 and 16 (C) 5, 2 and 8 (D) 5, 2 and 16



CatalyseR Eduventures (India) Pvt. Ltd.


REDOX REACTION 93

EXERCISE # 07 JEE (ADVANCED) CORNER

1. Consider a titration of potassium dichromate ssolution with acidified Mohr’s salt solution using
diphenylamine as indicator. The number of moles of Mohr’s salt required per mole of dichromate
is- [2007]
(A) 3 (B) 4 (C) 5 (D) 6

COMPREHENSION
Redox reactions play a pivotal role in chemistry and biology. The values of standard redox
potential  Eo  of two half-cell reactions decide which way the reaction is expected to proceed. A
simple example is a Daniel cell in which zinc goes into solution and copper gets deposited. Given
below are a set of half-cell reactions (acidic medium) along with their Eo (V with respect to
normal hydrogen electrode) values. Using this data obtain the correct explanations to Questions
39-41. [2007]

2. Among the following, identify the correct statement


(A) Chloride ion is oxidized by O2 (B) Fe2 is oxidized by iodine
(C) Iodide ion is oxidized by chlorine (D) Mn2  is oxidized by chlorine

3. While Fe3  is stable, Mn3  is not stable in acid solution because:


(A) O2 oxidises Mn2  to Mn3  (B) O2 oxidizes both Mn2  and Fe2 to Fe3 
(C) Fe3  oxidizes H2O to O2 (D) Mn3  oxidises H2O to O2

4. Sodium fusion extract, obtained from aniline, on treatment with iron (II) sulphate and H2 SO 4 in
presence of air gives a Prussian blue precipitate. The blue colour is due to the formation of-
(A) Fe4 Fe  CN6  (B) Fe3 Fe  CN6  (C) Fe4 Fe  CN6  (D) Fe3 Fe  CN6 
3 2 2 3

5. Match the reactions in Column I with nature of the reactions/type of the products in Column II.
Indicate your answer by darkening the appropriate bubbles of the 4 x 4 matrix given in ORS. [2007]
Column - I Column - II
(A) 
O  O2  O
2
2
2
(p) Redox reaction
(B) CrO24   H  (q) One of the products has trigonal planar
structure
(C) MnO4  NO2  H  (r) Dimeric bridged tetrahedral metal ion
(D) NO3  H2 SO4  Fe 2   (s) Disproportionation

Don’t ever get comfortable when you have the ability to achieve more.
94 REDOX REACTION

6. The oxidation number of Mn in the product of alkaline oxidative fusion of MnO 2 is: [2009]

7. The value of n in the molecular formula Ben Al2Si6 O18 is: [2010]

8. The total number of dirpotic acids among the following is - [2010]


H3PO4 H2 SO 4 H3PO3 H2 CO3 H2 S 2 O 7
H3BO3 H3PO2 H2 CrO 4 H2 SO3

9. The difference in the oxidation numbers of the two types of sulphur atoms in Na2 S 4 O6 is: [2011]

10. Reaction of Br2 with Na 2 CO3 in aqueous solution gives sodium bromide and sodium bromated
with evolution of CO2 gas. The number of sodium bromide molecules involved in the balanced
chemical equal is : [2011]

11. Oxidation states of the metal in the minerals haematite and magnetite, respectively, are [2011]
(A) II, III in haematite and III in magnetite (B) II, III in haematite and II in magnetite
(C) II in haematite and II, III in magnetite (D) III in haematite and II, III in magnetite

12. Reduction of the metal centre in aqueous permanganate ion involves [2011]
(A) 3 electrons in neutral medium (B) 5 electrons in neutral medium
(C) 3 electrons in alkaline medium (D) 5 electrons in acidic medium

13. Which ordering of compounds is according to the decreasing order of the oxidation state of
nitrogen? [2012]
(A) HNO3 ,NO,NH4 Cl,N2 (B) HNO3 ,NO,N2 ,NH4 Cl
(C) HNO3 ,NH4 Cl,NO,N2 (D) NO,HNO3 ,NH4 Cl,N2

14. The reaction of white phosphorous with aqueous NaOH gives phosphine along with another
phosphorus containing compound. The reaction type; the oxidation states of phosphorous in
phosphine and the other product are respectively [2012]
(A) redox reaction ; 3 and 5 (B) redox reaction ; 3 and 5
(C) disproportionation reaction ; 3 and 5 (D) disproportionation reaction ; 3 and 3

COMPREHENSION
Bleaching powder and bleach solution are produced on a large scale and used in several house
hold products. The effectiveness of bleach solution is often measured by iodometry [2012]

15. 25 mL of household bleach solution was mixed with 30 mL of 0.50 M KI and 10 mL of 4N acetic
acid. In the titration of the liberated iodine, 48 mL of 0.25 N Na 2S 2 O3 was used to reach the end
point. The molatiry of the household bleach solution is
(A) 0.48 M (B) 0.96 M (C) 0.24 M (D) 0.024 M

16. Bleaching powder contains a salt of an oxoacid as one of its components. The anhydride of that
oxoacid is:
(A) Cl2 O (B) Cl2O7 (C) ClO2 (D) Cl2O 6

CatalyseR Eduventures (India) Pvt. Ltd.


REDOX REACTION 95

17. Hydrogen peroxide in its reaction with KIO 4 and NH2 OH respectively, is acting as a - [2014]
(A) Reducing agent, oxidizing agent (B) Reducing agent, reducing agent
(C) Oxidising agent, oxidizing agent (D) Oxidising agent, reducing agent

18. For the reaction: [2014]


   
I  ClO  H2 SO 4  Cl  HSO  I2
3 4

The correct statement(s) in the balanced equation is/are:


(A) Stoichiometric coefficient of HSO 4 is 6. (B) Iodide is oxidized.
(C) Sulphur is reduced. (D) H2O is one of the products.

19. Consider the following list of reagents: [2014]


Acidified K 2Cr2 O7 , alkaline KMnO 4 ,CuSO 4 ,H2 O2 ,Cl2 ,O3 ,FeCl3 ,HNO 3 and Na2 S2 O3 . The total
number of reagents that can oxidize aqueous iodide to iodine is-

20. In neutral or faintly alkaline solution, 8 moles of permanganate anion quantitatively oxidize
thiosulphate anions to produce X moles of a sulphur containing product. The magnitude of X is:
[2016]


Don’t ever get comfortable when you have the ability to achieve more.
96 REDOX REACTION

ANSWER KEY
ANSWER KEY EXERCISE # 01 OBJECTIVE EXERCISE

Qs. 1 2 3 4 5 6 7 8 9 10
Ans. A A D C A B C B C D
Qs. 11 12 13 14 15 16 17 18 19 20
Ans. B B B B D C D A C D
Qs. 21 22 23 24 25 26 27 28 29 30
Ans. B D A C C B A B C A
Qs. 31 32 33 34 35
Ans. C D D D C

ANSWER KEY EXERCISE # 02 ADVANCED LEVEL EXERCISE

Qs. 1 2 3 4 5 6 7 8 9 10
Ans. CD ABC AB ABCD ACD ABD AC AB CD AC
Qs. 11 12 13 14 15 16 17 18 19 20
Ans. ABD BD AC AC AD C AC AC ABC ABC
Qs. 21 22 23 24 25
Ans. AC AB A D A  Q, S ; B  P;C  Q; D  R, S
Qs. 26 27
(A)  P,Q; (B)  R; (C)  S; (D)  Q,
Ans. (A)  Q; (B)  S; (C)  P; (D)  R
R,S
Qs. 28 29 30 31 32 33 34 35 36 37
Ans. A B A A B A B A C D
Qs. 38 39 40 41 42 43 44 45 46 47
Ans. C A 2 2 3 8 6 A A 4
Qs. 48 49 50
Ans. 3 2 5

ANSWER KEY EXERCISE # 03 SUBJECTIVE EXERCISE (LEVEL # 01)

Q.1 V = 168.54 ml Q.2 20.72 %


Q.3 V = 25 mL Q.4 MgCO3 = 51.22%, CaCO3 = 48.78 %
Q.5 83.33
Q.6 0.424 gm; 0.21gm Q.7 23.2 gm, 22.28gm Q.8 0.06gm; .0265gm
Q.9 4.24 g/L; 5.04 g/L Q.10 39.85%; 60.15%
Q.11 +3 Q.12 zero Q.13 V = 31.68 ml Q.14 0.588 N
Q.15 337 mL Q.16 41.53% Q.17 6.07  6 Q.18 0.15 N
Q.19 H2C2O4. 2H2O = 14.36%, KHC2O4. H2O = 81.71%
Q.20 1.406% Q.21 90.1% Q.22 27.27% Q.23 0.1281 g
Q.24 0.174g; 3.48% Q.25. 76.15%

CatalyseR Eduventures (India) Pvt. Ltd.


REDOX REACTION 97

ANSWER KEY EXERCISE # 04 ADVANCEDSUBJECTIVE EXERCISE (LEVEL # 02)

Q.1 Al = 0.546 g; Mg = 0.454 g Q.2 51.64%


Q.3 FeO = 13.34%; Fe2O3 = 86.66% Q.4 20 ml; 40 ml; 20 ml
Q.5 SCN+3I2+ 4H2O l SO42 + HCN + 7H+ + 6I, 15.68%
Q.6 –1 Q.7 30.55kg, 6 electrons
Q.8 5g Q.9 15.68% chromite
Q.10 VKIO3  0.42 mL, [HCl] = 0.0168N Q.11 57.4% CuS, 42.6% Cu2S
Q.12 4.67L Q.13 0.06 g
Q.14 1.76% Q.15 CuS = 57.3 %, 42.6%
Q.16 95.8 % Q.17 pyrite : 0.06 %; S = 0.032%
Q.18 40.77%
Q.19 59.16%
Q.20 X = 1, Y = 3, Z = 2, n = 2

ANSWER KEY EXERCISE # 05 KVPY, OLYMPIADS QUESTIONS

Qs. 1 2 3 4 5 6 7 8
Ans. B D D C C B B D

ANSWER KEY EXERCISE # 06 JEE (MAIN) CORNER

Qs. 1 2 3 4
Ans. A D B B

ANSWER KEY EXERCISE # 07 JEE (ADVANCED) CORNER

Qs. 1 2 3 4 5
Ans. D C D A (A)  ps, (B)  r (C)  p, q, (D)  p
Qs. 6 7 8 9 10 11 12 13 14 15
Ans. 6 3 6 5 5 D AD B C C
Qs. 16 17 18 19 20
Ans. A A ABD 7 6

Don’t ever get comfortable when you have the ability to achieve more.

You might also like